Neuro

Réussis tes devoirs et examens dès maintenant avec Quizwiz!

10. Which type of seizure is most likely to cause death for the patient? a. Subclinical seizures b. Myoclonic seizures c. Psychogenic seizures d. Tonic-clonic status epilepticus

10. d. Tonic-clonic status epilepticus is most dangerous because the continuous seizing can cause respiratory insufficiency, hypoxemia, cardiac dysrhythmia, hyperthermia, and systemic acidosis, which can all be fatal. Subclinical seizures may occur in a patient who is sedated, so there is no physical movement. Myoclonic seizures may occur in clusters and have a sudden, excessive jerk of the body that may hurl the person to the ground. Psychogenic seizures are psychiatric in origin and diagnosed with videoelectroencephalography (EEG) monitoring. They occur in patients with a history of emotional abuse or a specific traumatic episode.

13. The nurse teaches the patient taking antiseizure drugs that this method is most commonly used to measure compliance and monitor for toxicity. a. A daily seizure log b. Urine testing for drug levels c. Blood testing for drug levels d. Monthly electroencephalography (EEG)

13. c. Serum levels of antiseizure drugs are monitored regularly to maintain therapeutic levels of the drug, above which patients are likely to experience toxic effects and below which seizures are likely to occur. Many newer drugs do not require drug level monitoring because of large therapeutic ranges. A daily seizure log and urine testing for drug levels will not measure compliance or monitor for toxicity. EEGs have limited value in diagnosis of seizures and even less value in monitoring seizure control.

19. Which chronic neurologic disorder involves a deficiency of the neurotransmitters acetylcholine and γ aminobutyric acid (GABA) in the basal ganglia and extrapyramidal system? a. Myasthenia gravis b. Parkinson's disease c. Huntington's disease d. Amyotrophic lateral sclerosis (ALS)

19. c. Huntington's disease (HD) involves deficiency of acetylcholine and γ-aminobutyric acid (GABA) in the basal ganglia and extrapyramidal system that causes the opposite symptoms of parkinsonism. Myasthenia gravis involves autoimmune antibody destruction of cholinergic receptors at the neuromuscular junction. Amyotrophic lateral sclerosis (ALS) involves degeneration of motor neurons in the brainstem and spinal cord.

20. A 38-year-old woman has newly diagnosed multiple sclerosis (MS) and asks the nurse what is going to happen to her. What is the best response by the nurse? a. "You will have either periods of attacks and remissions or progression of nerve damage over time." b. "You need to plan for a continuous loss of movement, sensory functions, and mental capabilities." c. "You will most likely have a steady course of chronic progressive nerve damage that will change your personality." d. "It is common for people with MS to have an acute attack of weakness and then not to have any other symptoms for years."

20. a. Most patients with multiple sclerosis (MS) have remissions and exacerbations of neurologic dysfunction or a relapsing-remitting initial course followed by progression with or without occasional relapses, minor remissions, and plateaus that progressively cause loss of motor, sensory, and cerebellar functions. Intellectual function generally remains intact but patients may experience anger, depression, or euphoria. A few people have chronic progressive deterioration and some may experience only occasional and mild symptoms for several years after onset.

21. During assessment of a patient admitted to the hospital with an acute exacerbation of MS, what should the nurse expect to find? a. Tremors, dysphasia, and ptosis b. Bowel and bladder incontinence and loss of memory c. Motor impairment, visual disturbances, and paresthesias d. Excessive involuntary movements, hearing loss, and ataxia

21. c. Specific neurologic dysfunction of MS is caused by destruction of myelin and replacement with glial scar tissue at specific areas in the nervous system. Motor, sensory, cerebellar, and emotional dysfunctions, including paresthesias as well as patchy blindness, blurred vision, pain radiating along the dermatome of the nerve, ataxia, and severe fatigue, are the most common manifestations of MS. Constipation and bladder dysfunctions, short-term memory loss, sexual dysfunction, anger, and depression or euphoria may also occur. Excessive involuntary movements and tremors are not seen in MS.

23. Mitoxantrone (Novantrone) is being considered as treatment for a patient with progressive-relapsing MS. The nurse explains that a disadvantage of this drug compared with other drugs used for MS is what? a. It must be given subcutaneously every day. b. It has a lifetime dose limit because of cardiac toxicity. c. It is an anticholinergic agent that causes urinary incontinence. d. It is an immunosuppressant agent that increases the risk for infection.

23. b. Mitoxantrone (Novantrone) cannot be used for more than 2 to 3 years because it is an antineoplastic drug that causes cardiac toxicity, leukemia, and infertility. It is a monoclonal antibody given IV monthly when patients have inadequate responses to other drugs. It increases the risk of progressive multifocal leukoencephalopathy.

Priority Decision: A patient with MS has a nursing diagnosis of self-care deficit related to muscle spasticity and neuromuscular deficits. In providing care for the patient, what is most important for the nurse to do? a. Teach the family members how to care adequately for the patient's needs. b. Encourage the patient to maintain social interactions to prevent social isolation. c. Promote the use of assistive devices so the patient can participate in self-care activities. d. Perform all activities of daily living (ADLs) for the patient to conserve the patient's energy.

24. c. The main goal in care of the patient with MS is to keep the patient active and maximally functional and promote self-care as much as possible to maintain independence. Assistive devices encourage independence while preserving the patient's energy. No care activity that the patient can do for himself or herself should be performed by others. Involvement of the family in the patient's care and maintenance of social interactions are also important but are not the priority in care.

25. A patient with newly diagnosed MS has been hospitalized for evaluation and initial treatment of the disease. Following discharge teaching, the nurse realizes that additional instruction is needed when the patient says what? a. "It is important for me to avoid exposure to people with upper respiratory infections." b. "When I begin to feel better, I should stop taking the prednisone to prevent side effects." c. "I plan to use vitamin supplements and a high-protein diet to help manage my condition." d. "I must plan with my family how we are going to manage my care if I become more incapacitated."

25. b. Corticosteroids used in treating acute exacerbations of MS should not be abruptly stopped by the patient because adrenal insufficiency may result and prescribed tapering doses should be followed. Infections may exacerbate symptoms and should be avoided and high-protein diets with vitamin supplements are advocated. Long-term planning for increasing disability is also important.

26. The classic triad of manifestations associated with Parkinson's disease is tremor, rigidity, and bradykinesia. What is a consequence related to rigidity? a. Shuffling gait b. Impaired handwriting c. Lack of postural stability d. Muscle soreness and pain

26. d. The degeneration of dopamine-producing neurons in the substantia nigra of midbrain and basal ganglia lead to this triad of signs. Muscle soreness, pain, and slowness of movement are patient function consequences related to rigidity. Shuffling gait, lack of postural stability, absent arm swing while walking, absent blinking, masked facial expression, and difficulty initiating movement are all related to bradykinesia. Impaired handwriting and hand activities are related to the tremor of Parkinson's disease (PD).

27. A patient with a tremor is being evaluated for Parkinson's disease. The nurse explains to the patient that Parkinson's disease can be confirmed by a. CT and MRI scans. b. relief of symptoms with administration of dopaminergic agents. c. the presence of tremors that increase during voluntary movement. d. cerebral angiogram that reveals the presence of cerebral atherosclerosis.

27. b. Although clinical manifestations are characteristic in PD, no laboratory or diagnostic tests are specific for the condition. A diagnosis is made when at least two of the three signs of the classic triad are present and it is confirmed with a positive response to antiparkinsonian medication. Research regarding the role of genetic testing and MRI to diagnose PD is ongoing. Essential tremors increase during voluntary movement whereas the tremors of PD are more prominent at rest.

28. Which observation of the patient made by the nurse is most indicative of Parkinson's disease? a. Large, embellished handwriting b. Weakness of one leg resulting in a limping walk c. Difficulty rising from a chair and beginning to walk d. Onset of muscle spasms occurring with voluntary movement

28. c. The bradykinesia of PD prevents automatic movements and activities such as beginning to walk, rising from a chair, or even swallowing saliva cannot be executed unless they are consciously willed. Handwriting is affected by the tremor and results in the writing trailing off at the end of words. Specific limb weakness and muscle spasms are not characteristic of PD.

29. A patient with Parkinson's disease is started on levodopa. What should the nurse explain about this drug? a. It stimulates dopamine receptors in the basal ganglia. b. It promotes the release of dopamine from brain neurons. c. It is a precursor of dopamine that is converted to dopamine in the brain. d. It prevents the excessive breakdown of dopamine in the peripheral tissues.

29. c. Peripheral dopamine does not cross the blood-brain barrier but its precursor, levodopa, is able to enter the brain, where it is converted to dopamine, increasing the supply that is deficient in PD. Other drugs used to treat PD include bromocriptine, which stimulates dopamine receptors in the basal ganglia, and amantadine, which blocks the reuptake of dopamine into presynaptic neurons. Carbidopa is an agent that is usually administered with levodopa to prevent the levodopa from being metabolized in peripheral tissues before it can reach the brain.

31. A patient with myasthenia gravis is admitted to the hospital with respiratory insufficiency and severe weakness. When is a diagnosis of cholinergic crisis made? a. The patient's respiration is impaired because of muscle weakness. b. Administration of edrophonium (Tensilon) increases muscle weakness. c. Administration of edrophonium (Tensilon) results in improved muscle contractility. d. EMG reveals decreased response to repeated stimulation of muscles.

31. b. The reduction of the acetylcholine (ACh) effect in myasthenia gravis (MG) is treated with anticholinesterase drugs, which prolong the action of ACh at the neuromuscular synapse, but too much of these drugs will cause a cholinergic crisis with symptoms very similar to those of MG. To determine whether the patient's manifestations are due to a deficiency of ACh or to too much anticholinesterase drug, the anticholinesterase drug edrophonium chloride (Tensilon) is administered. If the patient is in cholinergic crisis, the patient's symptoms will worsen; if the patient is in a myasthenic crisis, the patient will improve.

Priority Decision: During care of a patient in myasthenic crisis, maintenance of what is the nurse's first priority for the patient? a. Mobility b. Nutrition c. Respiratory function d. Verbal communication

32. c. The patient in myasthenic crisis has severe weakness and fatigability of all skeletal muscles, affecting the patient's ability to breathe, swallow, talk, and move. However, the priority of nursing care is monitoring and maintaining adequate ventilation.

33. When providing care for a patient with ALS, the nurse recognizes what as one of the most distressing problems experienced by the patient? a. Painful spasticity of the face and extremities b. Retention of cognitive function with total degeneration of motor function c. Uncontrollable writhing and twisting movements of the face, limbs, and body d. Knowledge that there is a 50% chance the disease has been passed to any offspring

33. b. In ALS there is gradual degeneration of motor neurons with extreme muscle wasting from lack of stimulation and use. However, cognitive function is not impaired and patients feel trapped in a dying body. Chorea manifested by writhing, involuntary movements is characteristic of HD. As an autosomal dominant genetic disease, HD also has a 50% chance of being passed to each offspring.

34. In providing care for patients with chronic, progressive neurologic disease, what is the major goal of treatment that the nurse works toward? a. Meet the patient's personal care needs. b. Return the patient to normal neurologic function. c. Maximize neurologic functioning for as long as possible. d. Prevent the development of additional chronic diseases.

34. c. Many chronic neurologic diseases involve progressive deterioration in physical or mental capabilities and have no cure, with devastating results for patients and families. Health care providers can only attempt to alleviate physical symptoms, prevent complications, and assist patients in maximizing function and self-care abilities for as long as possible.

5. What is a nursing intervention that is appropriate for the patient with a nursing diagnosis of anxiety related to lack of knowledge of the etiology and treatment of headache? a. Help the patient to examine lifestyle patterns and precipitating factors. b. Administer medications as ordered to relieve pain and promote relaxation. c. Provide a quiet, dimly lit environment to reduce stimuli that increase muscle tension and anxiety. d. Support the patient's use of counseling or psychotherapy to enhance conflict resolution and stress reduction.

5. a. When the anxiety is related to a lack of knowledge about the etiology and treatment of a headache, helping the patient to identify stressful lifestyle patterns and other precipitating factors and ways of avoiding them are appropriate nursing interventions for the anxiety. Interventions that teach alternative therapies to supplement drug therapy also give the patient some control over pain and are appropriate teaching regarding treatment of the headache. The other interventions may help to reduce anxiety generally but they do not address the etiologic factor of the anxiety.

A patient with a suspected traumatic brain injury has bloody nasal drainage. What observation should cause the nurse to suspect that this patient has a cerebrospinal fluid (CSF) leak? A. A halo sign on the nasal drip pad B. Decreased blood pressure and urinary output C. A positive reading for glucose on a Test-tape strip D. Clear nasal drainage along with the bloody discharge

A

A 68-year-old man with suspected bacterial meningitis has just had a lumbar puncture in which cerebrospinal fluid was obtained for culture. Which medication should the nurse administer first? A. Codeine B. Phenytoin (Dilantin) C. Ceftriaxone (Rocephin) D. Acetaminophen (Tylenol)

A Bacterial meningitis is a medical emergency. When meningitis is suspected, antibiotic therapy (e.g., ceftriaxone) is instituted immediately after the collection of specimens for cultures, and even before the diagnosis is confirmed. Dexamethasone may also be prescribed before or with the first dose of antibiotics. The nurse should collaborate with the health care provider to manage the headache (with codeine), fever (with acetaminophen), and seizures (with phenytoin).

A 19-year-old woman is hospitalized for a frontal skull fracture from a blunt force head injury. Clear fluid is draining from the patient's nose. What action by the nurse is most appropriate? A. Apply a loose gauze pad under the patient's nose. B. Place the patient in a modified Trendelenburg position. C. Ask the patient to gently blow the nose to clear the drainage. D. Gently insert a catheter in the nares and suction the drainage.

A Cerebrospinal fluid (CSF) rhinorrhea (clear or bloody drainage from the nose) may occur with a frontal skull fracture. If CSF rhinorrhea occurs, the nurse should inform the physician immediately. A loose collection pad may be placed under the nose. The head of the bed may be raised to decrease the CSF pressure so that a tear can seal. The nurse should not place a dressing or tube in the nasal cavity, and the patient should not sneeze or blow the nose.

A 19-year-old woman is hospitalized for a frontal skull fracture from a blunt force head injury. Clear fluid is draining from the patient's nose. What action by the nurse is most appropriate? A. Apply a loose gauze pad under the patient's nose. B. Place the patient in a modified Trendelenburg position. C. Ask the patient to gently blow the nose to clear the drainage. D. Gently insert a catheter in the nares and suction the drainage.

A Cerebrospinal fluid (CSF) rhinorrhea (clear or bloody drainage from the nose) may occur with a frontal skull fracture. If CSF rhinorrhea occurs, the nurse should inform the physician immediately. A loose collection pad may be placed under the nose. The head of the bed may be raised to decrease the CSF pressure so that a tear can seal. The nurse should not place a dressing or tube in the nasal cavity, and the patient should not sneeze or blow the nose.

What nursing intervention should be implemented in the care of a patient who is experiencing increased ICP? A. Monitor fluid and electrolyte status carefully. B. Position the patient in a high Fowler's position. C. Administer vasoconstrictors to maintain cerebral perfusion. D. Maintain physical restraints to prevent episodes of agitation.

A Fluid and electrolyte disturbances can have an adverse effect on ICP and must be monitored vigilantly. The head of the patient's bed should be kept at 30 degrees in most circumstances, and physical restraints are not applied unless absolutely necessary. Vasoconstrictors are not typically administered in the treatment of ICP.

What nursing intervention should be implemented in the care of a patient who is experiencing increased ICP? A. Monitor fluid and electrolyte status carefully. B. Position the patient in a high Fowler's position. C. Administer vasoconstrictors to maintain cerebral perfusion. D. Maintain physical restraints to prevent episodes of agitation.

A Fluid and electrolyte disturbances can have an adverse effect on ICP and must be monitored vigilantly. The head of the patient's bed should be kept at 30 degrees in most circumstances, and physical restraints are not applied unless absolutely necessary. Vasoconstrictors are not typically administered in the treatment of ICP.

Magnetic resonance imaging (MRI) has revealed the presence of a brain tumor in a patient. The nurse should recognize that the patient will most likely need which treatment modality? A. Surgery B. Chemotherapy C. Radiation therapy D. Biologic drug therapy

A Surgical removal is the preferred treatment for brain tumors. Chemotherapy and biologic drug therapy are limited by the blood-brain barrier, tumor cell heterogeneity, and tumor cell drug resistance. Radiation therapy may be used as a follow-up measure after surgery.

Magnetic resonance imaging (MRI) has revealed the presence of a brain tumor in a patient. The nurse should recognize that the patient will most likely need which treatment modality? A. Surgery B. Chemotherapy C. Radiation therapy D. Biologic drug therapy

A Surgical removal is the preferred treatment for brain tumors. Chemotherapy and biologic drug therapy are limited by the blood-brain barrier, tumor cell heterogeneity, and tumor cell drug resistance. Radiation therapy may be used as a follow-up measure after surgery.

A patient with a suspected traumatic brain injury has bloody nasal drainage. What observation should cause the nurse to suspect that this patient has a cerebrospinal fluid (CSF) leak? A. A halo sign on the nasal drip pad B. Decreased blood pressure and urinary output C. A positive reading for glucose on a Test-tape strip D. Clear nasal drainage along with the bloody discharge

A When drainage containing both CSF and blood is allowed to drip onto a white pad, within a few minutes the blood will coalesce into the center, and a yellowish ring of CSF will encircle the blood, giving a halo effect. The presence of glucose would be unreliable for determining the presence of CSF because blood also contains glucose. Decreased blood pressure and urinary output would not be indicative of a CSF leak.

A 48-year-old man was just diagnosed with Huntington's disease. His 20-year-old son is upset about his father's diagnosis. How can the nurse best help this young man? A. Provide emotional and psychologic support. B. Encourage him to get diagnostic genetic testing done. C. Tell him the cognitive deterioration will be treated with counseling. D. Tell him the chorea and psychiatric disorders can be treated with haloperidol (Haldol).

A. The patient's son will first need emotional and psychologic support. He should be taught about diagnostic genetic testing for himself but should decide for himself with a genetic counselor if and when he wants this done. The treatment plan for his father will be determined depending on his father's needs.

The patient with type 1 diabetes mellitus with hypoglycemia is having a seizure. Which medication should the nurse anticipate administering to stop the seizure? A. IV dextrose solution B. IV diazepam (Valium) C. IV phenytoin (Dilantin) D. Oral carbamazepine (Tegretol)

A. This patient's seizure is caused by low blood glucose, so IV dextrose solution should be given first to stop the seizure. IV diazepam, IV phenytoin, and oral carbamazepine would be used to treat seizures from other causes such as head trauma, drugs, and infections.

A 64-year-old patient who has amyotrophic lateral sclerosis (ALS) is hospitalized with pneumonia. Which nursing action will be included in the plan of care? a. Assist with active range of motion (ROM). b. Observe for agitation and paranoia. c. Give muscle relaxants as needed to reduce spasms. d. Use simple words and phrases to explain procedures.

A. ALS causes progressive muscle weakness, but assisting the patient to perform active ROM will help maintain strength as long as possible. Psychotic manifestations such as agitation and paranoia are not associated with ALS. Cognitive function is not affected by ALS, and the patient's ability to understand procedures will not be impaired. Muscle relaxants will further increase muscle weakness and depress respirations.

A patient has ICP monitoring with an intraventricular catheter. A priority nursing intervention for the patient is a. aseptic technique to prevent infection b. constant monitoring of ICP waveforms c. removal of CSF to maintain normal ICP d. sampling CSF to determine abnormalities

A. Aseptic technique to prevent infection- An intraventricular catheter is a fluid coupled system that can provide direct access for microorganisms to enter the ventricles of the brain, and aseptic technique is a very high nursing priority to decrease the risk for infection. Constant monitoring of ICP waveforms is not usually necessary, and removal of CSF for sampling or to maintain normal ICP is done only when specifically ordered

After change-of-shift report, which patient should the nurse assess first? a. Patient with myasthenia gravis who is reporting increased muscle weakness b. Patient with a bilateral headache described as "like a band around my head" c. Patient with seizures who is scheduled to receive a dose of phenytoin (Dilantin) d. Patient with Parkinson's disease who has developed cogwheel rigidity of the arms

A. Because increased muscle weakness may indicate the onset of a myasthenic crisis, the nurse should assess this patient first. The other patients should also be assessed, but do not appear to need immediate nursing assessments or actions to prevent life-threatening complications.

A 31-year-old woman who has multiple sclerosis (MS) asks the nurse about risks associated with pregnancy. Which response by the nurse is accurate? a. "MS symptoms may be worse after the pregnancy." b. "Women with MS frequently have premature labor." c. "MS is associated with an increased risk for congenital defects." d. "Symptoms of MS are likely to become worse during pregnancy."

A. During the postpartum period, women with MS are at greater risk for exacerbation of symptoms. There is no increased risk for congenital defects in infants born of mothers with MS. Symptoms of MS may improve during pregnancy. Onset of labor is not affected by MS.

When a patient is admitted to the emergency department following a head injury, the nurse's first priority in management of the patient once a patent airway is confirmed is a. maintaining cervical spine precautions b. determining the presence of increased ICP c. monitoring for changes in neurologic status d. establishing IV access with a large-bore catheter

A. In addition to monitoring for a patent airway during emergency care of the patient with a head injury, the nurse must always assume that a patient with a head injury may have a cervical spine injury. Maintaining cervical spine precautions in all assessment and treatment activities with the patient is essential to prevent additional neurologic damage.

Which measure should the nurse prioritize when providing care for a patient with a diagnosis of multiple sclerosis (MS)? A. Vigilant infection control and adherence to standard precautions B. Careful monitoring of neurologic assessment and frequent reorientation C. Maintenance of a calorie count and hourly assessment of intake and output D. Assessment of blood pressure and monitoring for signs of orthostatic hypotension

A. Infection control is a priority in the care of patients with MS, since infection is the most common cause of an exacerbation of the disease. Decreases in cognitive function are less likely, and MS does not typically result in malnutrition, hypotension, or fluid volume excess or deficit.

The nurse advises a patient with myasthenia gravis (MG) to a. perform physically demanding activities early in the day. b. anticipate the need for weekly plasmapheresis treatments. c. do frequent weight-bearing exercise to prevent muscle atrophy. d. protect the extremities from injury due to poor sensory perception.

A. Muscles are generally strongest in the morning, and activities involving muscle activity should be scheduled then. Plasmapheresis is not routinely scheduled, but is used for myasthenia crisis or for situations in which corticosteroid therapy must be avoided. There is no decrease in sensation with MG, and muscle atrophy does not occur because although there is muscle weakness, they are still used.

A patient has a nursing diagnosis of risk for ineffective cerebral tissue perfusion related to cerebral edema. An appropriate nursing intervention for the patient is a. avoiding positioning the patient with neck and hip flexion b. maintaining hyperventilation to a PaCO2 of 15 to 20 mm Hg c. clustering nursing activities to provide periods of uninterrupted rest d. routine suctioning to prevent accumulation of respiratory secretions

A. Nursing care activities that increase ICP include hip and neck flexion, suctioning, clustering care activities, and noxious stimuli; they should be avoided or performed as little as possible in the patient with increased ICP. Lowering the PaCO2 below 20 mm Hg can cause ischemia and worsening of ICP; the PaCO2 should be maintained at 30 to 35 mm Hg.

A patient has been taking phenytoin (Dilantin) for 2 years. Which action will the nurse take when evaluating for adverse effects of the medication? a. Inspect the oral mucosa. b. Listen to the lung sounds. c. Auscultate the bowel tones. d. Check pupil reaction to light.

A. Phenytoin can cause gingival hyperplasia, but does not affect bowel tones, lung sounds, or pupil reaction to light.

The nurse observes a patient ambulating in the hospital hall when the patient's arms and legs suddenly jerk and the patient falls to the floor. The nurse will first a. assess the patient for a possible head injury. b. give the scheduled dose of divalproex (Depakote). c. document the timing and description of the seizure. d. notify the patient's health care provider about the seizure.

A. The patient who has had a myoclonic seizure and fall is at risk for head injury and should first be evaluated and treated for this possible complication. Documentation of the seizure, notification of the seizure, and administration of antiseizure medications are also appropriate actions, but the initial action should be assessment for injury.

For the patient undergoing a craniotomy, the nurse provides information about the use of wigs and hairpieces or other methods to disguise hair loss a. during pre operative teaching b. in the patient asks about their use c. in the immediate postoperative period d. when the patient expresses negative feelings about his or her appearance

A. The prevent undue concern and anxiety about hair loss and postoperative self-esteem disturbances, a patient undergoing cranial surgery should be informed pre operatively that the head is usually shaved in surgery while the patient is anesthetized and that methods can be used after the dressings are removed postoperatively to disguise the hair loss. In the immediate postoperative period, the patient is very ill, and the focus is on maintaining neurologic function, bur preoperatively the nurse should anticipate the patient's postoperative need for self-esteem and maintenance of appearance.

When assessing a patient with a traumatic brain injury, the nurse notes uncoordinated movement of the extremities. How should the nurse document this assessment? A. Ataxia B. Apraxia C. Anisocoria D. Anosognosia

A. Ataxia Ataxia is a lack of coordination of movement, possibly caused by lesions of sensory or motor pathways, cerebellum disorders, or certain medications. Apraxia is the inability to perform learned movements despite having the desire and physical ability to perform them related to a cerebral cortex lesion. Anisocoria is inequality of pupil size from an optic nerve injury. Anosognosia is the inability to recognize a bodily defect or disease related to lesions in the right parietal cortex.

A patient who has a neurologic disease that affects the pyramidal tract is likely to manifest what sign? A. Impaired muscle movement B. Decreased deep tendon reflexes C. Decreased level of consciousness D. Impaired sensation of touch, pain, and temperature

A. Impaired muscle movement Among the most important descending tracts are the corticobulbar and corticospinal tracts, collectively termed the pyramidal tract. These tracts carry volitional (voluntary) impulses from the cortex to the cranial and peripheral nerves. Dysfunction of the pyramidal tract is likely to manifest as impaired movement because of hypertonicity. Diseases affecting the pyramidal tract do not result in changes in LOC, impaired reflexes, or decreased sensation.

A 64-year-old patient who has amyotrophic lateral sclerosis (ALS) is hospitalized with pneumonia. Which nursing action will be included in the plan of care? a. Assist with active range of motion (ROM). b. Observe for agitation and paranoia. c. Give muscle relaxants as needed to reduce spasms. d. Use simple words and phrases to explain procedures.

ANS: A ALS causes progressive muscle weakness, but assisting the patient to perform active ROM will help maintain strength as long as possible. Psychotic manifestations such as agitation and paranoia are not associated with ALS. Cognitive function is not affected by ALS, and the patient's ability to understand procedures will not be impaired. Muscle relaxants will further increase muscle weakness and depress respirations. DIF: Cognitive Level: Apply (application) REF: 1439 TOP: Nursing Process: Planning MSC: NCLEX: Physiological Integrity

A hospitalized 31-year-old patient with a history of cluster headache awakens during the night with a severe stabbing headache. Which action should the nurse take first? a. Start the ordered PRN oxygen at 6 L/min. b. Put a moist hot pack on the patient's neck. c. Give the ordered PRN acetaminophen (Tylenol). d. Notify the patient's health care provider immediately.

ANS: A Acute treatment for cluster headache is administration of 100% oxygen at 6 to 8 L/min. If the patient obtains relief with the oxygen, there is no immediate need to notify the health care provider. Cluster headaches last only 60 to 90 minutes, so oral pain medications have minimal effect. Hot packs are helpful for tension headaches but are not as likely to reduce pain associated with a cluster headache. DIF: Cognitive Level: Apply (application) REF: 1417 OBJ: Special Questions: Prioritization TOP: Nursing Process: Implementation MSC: NCLEX: Physiological Integrity

A patient is brought to the emergency department (ED) by ambulance after being found unconscious on the bathroom floor by the spouse. Which action will the nurse take first? a. Obtain oxygen saturation. b. Check pupil reaction to light. c. Palpate the head for hematoma. d. Assess Glasgow Coma Scale (GCS).

ANS: A Airway patency and breathing are the most vital functions and should be assessed first. The neurologic assessments should be accomplished next and the health and medication history last. DIF: Cognitive Level: Application REF: 1435-1437

Which of these patients is most appropriate for the intensive care unit (ICU) charge nurse to assign to an RN who has floated from the medical unit? a. A 44-year-old receiving IV antibiotics for meningococcal meningitis b. A 23-year-old who had a skull fracture and craniotomy the previous day c. A 30-year-old who has an intracranial pressure (ICP) monitor in place after a head injury a week ago d. A 61-year-old who has increased ICP and is receiving hyperventilation therapy

ANS: A An RN who works on a medical unit will be familiar with administration of IV antibiotics and with meningitis. The postcraniotomy patient, patient with an ICP monitor, and the patient on a ventilator should be assigned to an RN familiar with the care of critically ill patients. DIF: Cognitive Level: Application REF: 1435-1438

When developing a plan of care for a patient with dysfunction of the cerebellum, the nurse will include interventions to a. prevent falls. b. stabilize mood. c. enhance swallowing ability. d. improve short-term memory.

ANS: A Because functions of the cerebellum include coordination and balance, the patient with dysfunction is at risk for falls. The cerebellum does not affect memory, mood, or swallowing ability. DIF: Cognitive Level: Application REF: 1410

After change-of-shift report, which patient should the nurse assess first? a. Patient with myasthenia gravis who is reporting increased muscle weakness b. Patient with a bilateral headache described as "like a band around my head" c. Patient with seizures who is scheduled to receive a dose of phenytoin (Dilantin) d. Patient with Parkinson's disease who has developed cogwheel rigidity of the arms

ANS: A Because increased muscle weakness may indicate the onset of a myasthenic crisis, the nurse should assess this patient first. The other patients should also be assessed, but do not appear to need immediate nursing assessments or actions to prevent life-threatening complications. DIF: Cognitive Level: Analyze (analysis) REF: 1438-1439 OBJ: Special Questions: Prioritization; Multiple Patients TOP: Nursing Process: Planning MSC: NCLEX: Safe and Effective Care Environment

When a patient's intracranial pressure (ICP) is being monitored with an intraventricular catheter, which information obtained by the nurse is most important to communicate to the health care provider? a. Oral temperature 101.6° F b. Apical pulse 102 beats/min c. Intracranial pressure 15 mm Hg d. Mean arterial pressure 90 mm Hg

ANS: A Infection is a serious consideration with ICP monitoring, especially with intraventricular catheters. The temperature indicates the need for antibiotics or removal of the monitor. The ICP, arterial pressure, and apical pulse are all borderline high but require only ongoing monitoring at this time. DIF: Cognitive Level: Application REF: 1438-1440

he nurse advises a patient with myasthenia gravis (MG) to a. perform physically demanding activities early in the day. b. anticipate the need for weekly plasmapheresis treatments. c. do frequent weight-bearing exercise to prevent muscle atrophy. d. protect the extremities from injury due to poor sensory perception.

ANS: A Muscles are generally strongest in the morning, and activities involving muscle activity should be scheduled then. Plasmapheresis is not routinely scheduled, but is used for myasthenia crisis or for situations in which corticosteroid therapy must be avoided. There is no decrease in sensation with MG, and muscle atrophy does not occur because although there is muscle weakness, they are still used. DIF: Cognitive Level: Apply (application) REF: 1437 TOP: Nursing Process: Implementation MSC: NCLEX: Physiological Integrity

After noting that a patient with a head injury has clear nasal drainage, which action should the nurse take? a. Have the patient blow the nose. b. Check the nasal drainage for glucose. c. Assure the patient that rhinorrhea is normal after a head injury. d. Obtain a specimen of the fluid to send for culture and sensitivity.

ANS: B Clear nasal drainage in a patient with a head injury suggests a dural tear and cerebrospinal fluid (CSF) leakage. If the drainage is CSF, it will test positive for glucose. Fluid leaking from the nose will have normal nasal flora, so culture and sensitivity will not be useful. Blowing the nose is avoided to prevent CSF leakage. DIF: Cognitive Level: Application REF: 1438-1439

A patient who has bacterial meningitis is disoriented and anxious. Which nursing action will be included in the plan of care? a. Encourage family members to remain at the bedside. b. Apply soft restraints to protect the patient from injury. c. Keep the room well-lighted to improve patient orientation. d. Minimize contact with the patient to decrease sensory input.

ANS: A Patients with meningitis and disorientation will be calmed by the presence of someone familiar at the bedside. Restraints should be avoided because they increase agitation and anxiety. The patient requires frequent assessment for complications; the use of touch and a soothing voice will decrease anxiety for most patients. The patient will have photophobia, so the light should be dim. DIF: Cognitive Level: Application REF: 1453-1455

A patient has been taking phenytoin (Dilantin) for 2 years. Which action will the nurse take when evaluating for adverse effects of the medication? a. Inspect the oral mucosa. b. Listen to the lung sounds. c. Auscultate the bowel tones. d. Check pupil reaction to light.

ANS: A Phenytoin can cause gingival hyperplasia, but does not affect bowel tones, lung sounds, or pupil reaction to light. DIF: Cognitive Level: Apply (application) REF: 1424 TOP: Nursing Process: Evaluation MSC: NCLEX: Physiological Integrity

While admitting a patient with a basal skull fracture, the nurse notes clear drainage from the patient's nose. Which of these admission orders should the nurse question? a. Insert nasogastric tube. b. Turn patient every 2 hours. c. Keep the head of bed elevated. d. Apply cold packs for facial bruising.

ANS: A Rhinorrhea may indicate a dural tear with cerebrospinal fluid (CSF) leakage, and insertion of a nasogastric tube will increase the risk for infections such as meningitis. Turning the patient, elevating the head, and applying cold pack are appropriate orders. DIF: Cognitive Level: Application REF: 1440

The charge nurse observes an inexperienced staff nurse who is caring for a patient who has had a craniotomy for a brain tumor. Which action by the inexperienced nurse requires the charge nurse to intervene? a. The staff nurse suctions the patient every 2 hours. b. The staff nurse assesses neurologic status every hour. c. The staff nurse elevates the head of the bed to 30 degrees. d. The staff nurse administers a mild analgesic before turning the patient.

ANS: A Suctioning increases intracranial pressure and is done only when the patient's respiratory condition indicates it is needed. The other actions by the staff nurse are appropriate. DIF: Cognitive Level: Application REF: 1430-1431

A patient with a head injury has admission vital signs of blood pressure 128/68, pulse 110, and respirations 26. Which of these vital signs, if taken 1 hour after admission, will be of most concern to the nurse? a. Blood pressure 156/60, pulse 55, respirations 12 b. Blood pressure 130/72, pulse 90, respirations 32 c. Blood pressure 148/78, pulse 112, respirations 28 d. Blood pressure 110/70, pulse 120, respirations 30

ANS: A Systolic hypertension with widening pulse pressure, bradycardia, and respiratory changes represent Cushing's triad and indicate that the intracranial pressure (ICP) has increased, and brain herniation may be imminent unless immediate action is taken to reduce ICP. The other vital signs may indicate the need for changes in treatment, but they are not indicative of an immediately life-threatening process. DIF: Cognitive Level: Application REF: 1429-1430

When caring for a patient who has had a head injury, which assessment information requires the most rapid action by the nurse? a. The patient is more difficult to arouse. b. The patient's pulse is slightly irregular. c. The patient's blood pressure increases from 120/54 to 136/62 mm Hg. d. The patient complains of a headache at pain level 5 of a 10-point scale.

ANS: A The change in level of consciousness (LOC) is an indicator of increased intracranial pressure (ICP) and suggests that action by the nurse is needed to prevent complications. The change in BP should be monitored but is not an indicator of a need for immediate nursing action. Headache is not unusual in a patient after a head injury. A slightly irregular apical pulse is not unusual. DIF: Cognitive Level: Application REF: 1431-1433

When admitting a patient who has a tumor of the right frontal lobe, the nurse would expect to find a. judgment changes. b. expressive aphasia. c. right-sided weakness. d. difficulty swallowing.

ANS: A The frontal lobes control intellectual activities such as judgment. Speech is controlled in the parietal lobe. Weakness and hemiplegia occur on the contralateral side from the tumor. Swallowing is controlled by the brainstem. DIF: Cognitive Level: Application REF: 1447 | 1448

A 46-year-old patient tells the nurse about using acetaminophen (Tylenol) several times every day for recurrent bilateral headaches. Which action will the nurse plan to take first? a. Discuss the need to stop taking the acetaminophen. b. Suggest the use of biofeedback for headache control. c. Describe the use of botulism toxin (Botox) for headaches. d. Teach the patient about magnetic resonance imaging (MRI).

ANS: A The headache description suggests that the patient is experiencing medication overuse headache. The initial action will be withdrawal of the medication. The other actions may be needed if the headaches persist. DIF: Cognitive Level: Apply (application) REF: 1418 OBJ: Special Questions: Prioritization TOP: Nursing Process: Planning MSC: NCLEX: Physiological Integrity

The nurse obtains these assessment findings for a patient who has a head injury. Which finding should be reported rapidly to the health care provider? a. Urine output of 800 mL in the last hour b. Intracranial pressure of 16 mm Hg when patient is turned c. Ventriculostomy drains 10 mL of cerebrospinal fluid per hour d. LICOX brain tissue oxygenation catheter shows PbtO2 of 38 mm Hg

ANS: A The high urine output indicates that diabetes insipidus may be developing and interventions to prevent dehydration need to be rapidly implemented. The other data do not indicate a need for any change in therapy. DIF: Cognitive Level: Application REF: 1434-1435

The nurse observes a patient ambulating in the hospital hall when the patient's arms and legs suddenly jerk and the patient falls to the floor. The nurse will first a. assess the patient for a possible head injury. b. give the scheduled dose of divalproex (Depakote). c. document the timing and description of the seizure. d. notify the patient's health care provider about the seizure.

ANS: A The patient who has had a myoclonic seizure and fall is at risk for head injury and should first be evaluated and treated for this possible complication. Documentation of the seizure, notification of the seizure, and administration of antiseizure medications are also appropriate actions, but the initial action should be assessment for injury. DIF: Cognitive Level: Apply (application) REF: 1423 OBJ: Special Questions: Prioritization TOP: Nursing Process: Implementation MSC: NCLEX: Physiological Integrity

An unconscious patient has a nursing diagnosis of ineffective cerebral tissue perfusion related to cerebral tissue swelling. Which nursing intervention will be included in the plan of care? a. Keep the head of the bed elevated to 30 degrees. b. Position the patient with the knees and hips flexed. c. Encourage coughing and deep breathing to improve oxygenation. d. Cluster nursing interventions to provide uninterrupted rest periods.

ANS: A The patient with increased intracranial pressure (ICP) should be maintained in the head-up position to help reduce ICP. Flexion of the hips and knees increases abdominal pressure, which increases ICP. Because the stimulation associated with nursing interventions increases ICP, clustering interventions will progressively elevate ICP. Coughing increases intrathoracic pressure and ICP. DIF: Cognitive Level: Application REF: 1436-1437

To assess the functioning of the trigeminal and facial nerves (CN V and VII), the nurse should a. apply a cotton wisp strand to the cornea. b. have the patient read a magazine or book. c. shine a bright light into the patient's pupil. d. check for unilateral drooping of the eyelids.

ANS: A The trigeminal and facial nerves are responsible for the corneal reflex. The optic nerve is tested by having the patient read a Snellen chart or a newspaper. Assessment of pupil response to light and ptosis are used to check function of the oculomotor nerve. DIF: Cognitive Level: Comprehension REF: 1416-1417

When admitting a patient with a possible coup-contracoup injury after a car accident to the emergency department, the nurse obtains the following information. Which finding is most important to report to the health care provider? a. The patient takes warfarin (Coumadin) daily. b. The patient's blood pressure is 162/94 mm Hg. c. The patient is unable to remember the accident. d. The patient complains of a severe dull headache.

ANS: A The use of anticoagulants increases the risk for intracranial hemorrhage and should be immediately reported. The other information would not be unusual in a patient with a head injury who had just arrived to the ED. DIF: Cognitive Level: Application REF: 1437-1438

When admitting an acutely confused patient with a head injury, which action should the nurse take? a. Ask family members about the patient's health history. b. Ask leading questions to assist in obtaining health data. c. Wait until the patient is better oriented to ask questions. d. Obtain only the physiologic neurologic assessment data.

ANS: A When admitting a patient who is likely to be a poor historian, the nurse should obtain health history information from others who have knowledge about the patient's health. Waiting until the patient is oriented or obtaining only physiologic data will result in incomplete assessment data; this could adversely affect decision-making about treatment. Asking leading questions may result in inaccurate or incomplete information. DIF: Cognitive Level: Application REF: 1412-1413

charge nurse is observing a new staff nurse who is assessing a patient with a possible spinal cord lesion for sensation. Which action indicates a need for further teaching about neurologic assessment? a. The new nurse asks the patient, "Does this feel sharp?" b. The new nurse tests for light touch before testing for pain. c. The new nurse has the patient close the eyes during testing. d. The new nurse uses an irregular pattern to test for intact touch.

ANS: A When performing a sensory assessment, the nurse should not provide verbal clues. The other actions by the new nurse are appropriate. DIF: Cognitive Level: Application REF: 1417-1418

A patient who has a head injury is diagnosed with a concussion. Which action will the nurse plan to take? a. Coordinate the transfer of the patient to the operating room. b. Provide discharge instructions about monitoring neurologic status. c. Transport the patient to radiology for magnetic resonance imaging (MRI) of the brain. d. Arrange to admit the patient to the neurologic unit for observation for 24 hours.

ANS: B A patient with a minor head trauma is usually discharged with instructions about neurologic monitoring and the need to return if neurologic status deteriorates. MRI, hospital admission, or surgery are not indicated in a patient with a concussion. DIF: Cognitive Level: Application REF: 1440

The nurse notes in the patient's medical history that the patient has a positive Romberg test. Which nursing diagnosis is appropriate? a. Acute pain related to hyperreflexia and spasm b. Risk for falls related to dizziness or weakness c. Disturbed tactile sensory perception related to spinal cord damage d. Ineffective thermoregulation related to decreased vasomotor response

ANS: B A positive Romberg test indicates that the patient has difficulty maintaining balance with the eyes closed. The Romberg does not test for tactile perception, thermoregulation, or hyperreflexia. DIF: Cognitive Level: Application REF: 1417-1418

The following orders are received for an unconscious patient who has just arrived in the emergency department after a head injury caused by an automobile accident. Which one should the nurse question? a. Obtain x-rays of the skull and spine. b. Prepare the patient for lumbar puncture. c. Send for computed tomography (CT) scan. d. Perform neurologic checks every 15 minutes.

ANS: B After a head injury, the patient may be experiencing intracranial bleeding and increased intracranial pressure, which could lead to herniation of the brain with lumbar puncture. The other orders are appropriate. DIF: Cognitive Level: Application REF: 1418-1419

Which information about a 71-year-old patient is most important for the admitting nurse to report to the patient's health care provider? a. Triceps reflex response graded at 1/5 b. Recent unintended weight loss of 20 pounds c. Patient complaint of chronic difficulty in falling asleep d. Orthostatic drop in systolic blood pressure of 10 mm Hg

ANS: B Although changes in appetite are normal with aging, a 20-pound weight loss requires further investigation. Orthostatic drops in blood pressure, changes in sleep patterns, and slowing of reflexes are normal changes in aging. DIF: Cognitive Level: Application REF: 1412-1413 | 1414

A patient has a lesion that affects lower motor neurons. During assessment of the patient's lower extremities, the nurse expects to find a. spasticity. b. flaccidity. c. loss of sensation. d. hyperactive reflexes.

ANS: B Because the cell bodies of lower motor neurons are located in the spinal cord, damage to the neuron will decrease motor activity of the affected muscles. Spasticity and hyperactive reflexes are caused by upper motor neuron damage. Sensation is not impacted by motor neuron lesions. DIF: Cognitive Level: Comprehension REF: 1408-1409

Which assessment information will the nurse collect to determine whether a patient is developing postconcussion syndrome? a. Muscle resistance b. Short-term memory c. Glasgow coma scale d. Pupil reaction to light

ANS: B Decreased short-term memory is one indication of postconcussion syndrome. The other data may be assessed but are not indications of postconcussion syndrome. DIF: Cognitive Level: Application REF: 1440

Which statement by a patient who is being discharged from the emergency department (ED) after a head injury indicates a need for intervention by the nurse? a. "I will return if I feel dizzy or nauseated." b. "I am going to drive home and go to bed." c. "I do not even remember being in an accident." d. "I can take acetaminophen (Tylenol) for my headache."

ANS: B Following a head injury, the patient should avoid operating heavy machinery. Retrograde amnesia is common after a concussion. The patient can take acetaminophen for headache and should return if symptoms of increased intracranial pressure such as dizziness or nausea occur. DIF: Cognitive Level: Application REF: 1444

A patient is scheduled for a lumbar puncture. The nurse will plan to a. transfer the patient to radiology just before the procedure. b. help the patient to a side lying position before the procedure. c. place the patient on NPO status for 4 hours before the procedure. d. administer a sedative medication 30 minutes before the procedure.

ANS: B For a lumbar puncture, the patient lies in the lateral recumbent position. The procedure does not usually require a sedative, is done in the patient room, and has no risk for aspiration. DIF: Cognitive Level: Application REF: 1418-1419

A patient is scheduled for a myelogram to confirm the presence of a herniated intervertebral disk. Which information obtained when admitting the patient is most important for the nurse to communicate to the health care provider before the procedure? a. The patient is anxious about the test. b. The patient has an allergy to shellfish. c. The patient had 4 ounces of apple juice 4 hours earlier. d. The patient has back pain when lying flat for long periods.

ANS: B Iodine-containing contrast medium is injected into the subarachnoid space during a myelogram. The health care provider may need to modify the postmyelogram orders to prevent back pain, but this can be done after the procedure. Clear liquids are usually considered safe up to 4 hours before a diagnostic or surgical procedure. The patient's anxiety should be addressed, but this is not as important as the iodine allergy. DIF: Cognitive Level: Application REF: 1421-1422

The home health registered nurse (RN) is planning care for a patient with a seizure disorder related to a recent head injury. Which nursing action can be delegated to a licensed practical/vocational nurse (LPN/LVN)? a. Make referrals to appropriate community agencies. b. Place medications in the home medication organizer. c. Teach the patient and family how to manage seizures. d. Assess for use of medications that may precipitate seizures.

ANS: B LPN/LVN education includes administration of medications. The other activities require RN education and scope of practice. DIF: Cognitive Level: Apply (application) REF: 1426 OBJ: Special Questions: Delegation TOP: Nursing Process: Planning MSC: NCLEX: Safe and Effective Care Environment

Which intervention will the nurse include in the plan of care for a patient with primary restless legs syndrome (RLS) who is having difficulty sleeping? a. Teach about the use of antihistamines to improve sleep. b. Suggest that the patient exercise regularly during the day. c. Make a referral to a massage therapist for deep massage of the legs. d. Assure the patient that the problem is transient and likely to resolve.

ANS: B Nondrug interventions such as getting regular exercise are initially suggested to improve sleep quality in patients with RLS. Antihistamines may aggravate RLS. Massage does not alleviate RLS symptoms and RLS is likely to progress in most patients. DIF: Cognitive Level: Apply (application) REF: 1427 TOP: Nursing Process: Planning MSC: NCLEX: Physiological Integrity

When family members ask the nurse about the purpose of the ventriculostomy system being used for intracranial pressure monitoring for a patient, which response by the nurse is best? a. "This type of monitoring system is complex and highly skilled staff are needed." b. "The monitoring system helps show whether blood flow to the brain is adequate." c. "The ventriculostomy monitoring system helps check for alterations in cerebral perfusion pressure." d. "This monitoring system has multiple benefits including facilitation of cerebrospinal fluid drainage."

ANS: B Short and simple explanations should be given to patients and family members. The other explanations are either too complicated to be easily understood or may increase the family member's anxiety. DIF: Cognitive Level: Application REF: 1438

After suctioning, the nurse notes that the intracranial pressure for a patient with a traumatic head injury has increased from 14 to 16 mm Hg. Which action should the nurse take first? a. Document the increase in intracranial pressure. b. Assure that the patient's neck is not in a flexed position. c. Notify the health care provider about the change in pressure. d. Increase the rate of the prescribed propofol (Diprovan) infusion.

ANS: B Since suctioning will cause a transient increase in intracranial pressure, the nurse should initially check for other factors that might be contributing to the increase and observe the patient for a few minutes. Documentation is needed, but this is not the first action. There is no need to notify the health care provider about this expected reaction to suctioning. Propofol is used to control patient anxiety or agitation; there is no indication that anxiety has contributed to the increase in intracranial pressure. DIF: Cognitive Level: Application REF: 1426 | 1435-1437 | 1436-1437

When a 74-year-old patient is seen in the health clinic with new development of a stooped posture, shuffling gait, and pill rolling-type tremor, the nurse will anticipate teaching the patient about a. oral corticosteroids. b. antiparkinsonian drugs. c. magnetic resonance imaging (MRI). d. electroencephalogram (EEG) testing.

ANS: B The diagnosis of Parkinson's is made when two of the three characteristic manifestations of tremor, rigidity, and bradykinesia are present. The confirmation of the diagnosis is made on the basis of improvement when antiparkinsonian drugs are administered. This patient has symptoms of tremor and bradykinesia. The next anticipated step will be treatment with medications. MRI and EEG are not useful in diagnosing Parkinson's disease, and corticosteroid therapy is not used to treat it. DIF: Cognitive Level: Apply (application) REF: 1434 TOP: Nursing Process: Planning MSC: NCLEX: Physiological Integrity

Neurologic testing of the patient indicates impaired functioning of the left glossopharyngeal nerve (CN IX) and the vagus nerve (CN X). Which action will the nurse include in the plan of care? a. Insert an oral airway. b. Withhold oral fluid or foods. c. Provide highly seasoned foods. d. Apply artificial tears every hour.

ANS: B The glossopharyngeal and vagus nerves innervate the pharynx and control the gag reflex; a patient with impaired function of these nerves is at risk for aspiration. An oral airway may be needed when a patient is unconscious and unable to maintain the airway, but it will not decrease aspiration risk. Taste and eye blink are controlled by the facial nerve. DIF: Cognitive Level: Application REF: 1417

Following a head injury, an unconscious 32-year-old patient is admitted to the emergency department (ED). The patient's spouse and children stay at the patient's side and constantly ask about the treatment being given. What action is best for the nurse to take? a. Ask the family to stay in the waiting room until the initial assessment is completed. b. Allow the family to stay with the patient and briefly explain all procedures to them. c. Call the family's pastor or spiritual advisor to support them while initial care is given. d. Refer the family members to the hospital counseling service to deal with their anxiety.

ANS: B The need for information about the diagnosis and care is very high in family members of acutely ill patients, and the nurse should allow the family to observe care and explain the procedures. A pastor or counseling service can offer some support, but research supports information as being more effective. Asking the family to stay in the waiting room will increase their anxiety. DIF: Cognitive Level: Application REF: 1438

A patient with a head injury opens the eyes to verbal stimulation, curses when stimulated, and does not respond to a verbal command to move but attempts to remove a painful stimulus. The nurse records the patient's Glasgow Coma Scale score as a. 9. b. 11. c. 13. d. 15.

ANS: B The patient has a score of 3 for eye opening, 3 for best verbal response, and 5 for best motor response. DIF: Cognitive Level: Application REF: 1434

ollowing a thymectomy, a 62-year-old male patient with myasthenia gravis receives the usual dose of pyridostigmine (Mestinon). An hour later, the patient complains of nausea and severe abdominal cramps. Which action should the nurse take first? a. Auscultate the patient's bowel sounds. b. Notify the patient's health care provider. c. Administer the prescribed PRN antiemetic drug. d. Give the scheduled dose of prednisone (Deltasone).

ANS: B The patient's history and symptoms indicate a possible cholinergic crisis. The health care provider should be notified immediately, and it is likely that atropine will be prescribed. The other actions will be appropriate if the patient is not experiencing a cholinergic crisis. DIF: Cognitive Level: Apply (application) REF: 1438-1439 OBJ: Special Questions: Prioritization TOP: Nursing Process: Implementation MSC: NCLEX: Physiological Integrity

A patient with possible cerebral edema has a serum sodium level of 115 mEq/L (115 mmol/L) and a decreasing level of consciousness (LOC) and complains of a headache. Which of these prescribed interventions should the nurse implement first? a. Draw blood for arterial blood gases (ABGs). b. Administer 5% hypertonic saline intravenously. c. Administer acetaminophen (Tylenol) 650 mg orally. d. Send patient for computed tomography (CT) of the head.

ANS: B The patient's low sodium indicates that hyponatremia may be causing the cerebral edema, and the nurse's first action should be to correct the low sodium level. Acetaminophen (Tylenol) will have minimal effect on the headache because it is caused by cerebral edema and increased intra-cranial pressure (ICP). Drawing ABGs and obtaining a CT scan may add some useful information, but the low sodium level may lead to seizures unless it is addressed quickly. DIF: Cognitive Level: Application REF: 1452-1455

A hospitalized patient complains of a bilateral headache, 4/10 on the pain scale, that radiates from the base of the skull. Which prescribed PRN medications should the nurse administer initially? a. Lorazepam (Ativan) b. Acetaminophen (Tylenol) c. Morphine sulfate (Roxanol) d. Butalbital and aspirin (Fiorinal)

ANS: B The patient's symptoms are consistent with a tension headache, and initial therapy usually involves a nonopioid analgesic such as acetaminophen, which is sometimes combined with a sedative or muscle relaxant. Lorazepam may be used in conjunction with acetaminophen but would not be appropriate as the initial monotherapy. Morphine sulfate and butalbital and aspirin would be more appropriate for a headache that did not respond to a nonopioid analgesic. DIF: Cognitive Level: Apply (application) REF: 1414 OBJ: Special Questions: Prioritization TOP: Nursing Process: Implementation MSC: NCLEX: Physiological Integrity

The health care provider is considering the use of sumatriptan (Imitrex) for a 54-year-old male patient with migraine headaches. Which information obtained by the nurse is most important to report to the health care provider? a. The patient drinks 1 to 2 cups of coffee daily. b. The patient had a recent acute myocardial infarction. c. The patient has had migraine headaches for 30 years. d. The patient has taken topiramate (Topamax) for 2 months.

ANS: B The triptans cause coronary artery vasoconstriction and should be avoided in patients with coronary artery disease. The other information will be reported to the health care provider, but none of it indicates that sumatriptan would be an inappropriate treatment. DIF: Cognitive Level: Apply (application) REF: 1416 OBJ: Special Questions: Prioritization TOP: Nursing Process: Assessment MSC: NCLEX: Physiological Integrity

The nurse will assess a 67-year-old patient who is experiencing a cluster headache for a. nuchal rigidity. b. unilateral ptosis. c. projectile vomiting. d. throbbing, bilateral facial pain.

ANS: B Unilateral eye edema, tearing, and ptosis are characteristic of cluster headaches. Nuchal rigidity suggests meningeal irritation, such as occurs with meningitis. Although nausea and vomiting may occur with migraine headaches, projectile vomiting is more consistent with increased intracranial pressure (ICP). Unilateral sharp, stabbing pain, rather than throbbing pain, is characteristic of cluster headaches. DIF: Cognitive Level: Understand (comprehension) REF: 1414 TOP: Nursing Process: Assessment MSC: NCLEX: Physiological Integrity

7. When obtaining a health history and physical assessment for a 36-year-old female patient with possible multiple sclerosis (MS), the nurse should a. assess for the presence of chest pain. b. inquire about urinary tract problems. c. inspect the skin for rashes or discoloration. d. ask the patient about any increase in libido.

ANS: B Urinary tract problems with incontinence or retention are common symptoms of MS. Chest pain and skin rashes are not symptoms of MS. A decrease in libido is common with MS. DIF: Cognitive Level: Apply (application) REF: 1429 TOP: Nursing Process: Assessment MSC: NCLEX: Physiological Integrity

Which equipment will the nurse obtain to assess vibration sense in a patient who has peripheral nerve dysfunction? a. Sharp pin b. Tuning fork c. Reflex hammer d. Calibrated compass

ANS: B Vibration sense is testing by touching the patient with a vibrating tuning fork. The other equipment is needed for testing of pain sensation, reflexes, and two-point discrimination. DIF: Cognitive Level: Comprehension REF: 1417-1419

A patient admitted with bacterial meningitis and a temperature of 102° F (38.8° C) has orders for all of these collaborative interventions. Which action should the nurse take first? a. Administer ceftizoxime (Cefizox) 1 g IV. b. Use a cooling blanket to lower temperature. c. Swap the nasopharyngeal mucosa for cultures. d. Give acetaminophen (Tylenol) 650 mg PO.

ANS: C Antibiotic therapy should be instituted rapidly in bacterial meningitis, but cultures must be done before antibiotics are started. As soon as the cultures are done, the antibiotic should be started. Hypothermia therapy and acetaminophen administration are appropriate but can be started after the other actions are implemented. DIF: Cognitive Level: Application REF: 1440-1441

Which assessment is most important for the nurse to make regarding a patient with myasthenia gravis? a. Pupil size b. Grip strength c. Respiratory effort d. Level of consciousness

ANS: C Because respiratory insufficiency may be life threatening, it will be most important to monitor respiratory function. The other data also will be assessed but are not as critical. DIF: Cognitive Level: Apply (application) REF: 1438-1439 OBJ: Special Questions: Prioritization TOP: Nursing Process: Assessment MSC: NCLEX: Physiological Integrity

Propranolol (Inderal), a β-adrenergic blocker that inhibits sympathetic nervous system activity, is prescribed for a patient. The nurse monitors the patient for a. dry mouth. b. constipation. c. slowed pulse. d. urinary retention.

ANS: C Inhibition of the fight or flight response leads to decreased heart rate. Dry mouth, constipation, and urinary retention are associated with peripheral nervous system blockade. DIF: Cognitive Level: Comprehension REF: 1407

When the nurse applies a painful stimulus to the nail beds of an unconscious patient, the patient responds with internal rotation, adduction, and flexion of the arms. The nurse documents this as a. flexion withdrawal. b. localization of pain. c. decorticate posturing. d. decerebrate posturing.

ANS: C Internal rotation, adduction, and flexion of the arms in an unconscious patient is documented as decorticate posturing. Extension of the arms and legs is decerebrate posturing. Because the flexion is generalized, it does not indicate localization of pain or flexion withdrawal. DIF: Cognitive Level: Comprehension REF: 1429-1430

While caring for a patient who has just been admitted with meningococcal meningitis, the RN observes all of the following. Which one requires action by the RN? a. The bedrails at the head and foot of the bed are both elevated. b. The patient receives a regular diet from the dietary department. c. The nursing assistant goes into the patient's room without a mask. d. The lights in the patient's room are turned off and the blinds are shut.

ANS: C Meningococcal meningitis is spread by respiratory secretions, so it is important to maintain respiratory isolation as well as standard precautions. Because the patient may be confused and weak, bedrails should be elevated at both the food and head of the bed. Low light levels in the room decrease pain caused by photophobia. Nutrition is an important aspect of care in a patient with meningitis. DIF: Cognitive Level: Application REF: 1453-1455

When caring for a patient who has had cerebral angiography, which nursing action will be included in the plan of care? a. Ask about headache and photophobia. b. Keep patient NPO until gag reflex returns. c. Check pulse and blood pressure frequently. d. Assess orientation to person, place, and time.

ANS: C Since a catheter is inserted into an artery (such as the femoral artery) during cerebral angiography, the nurse should assess for bleeding after this procedure. The other nursing assessments are not necessary after angiography. DIF: Cognitive Level: Application REF: 1418-1419

The community health nurse is developing a program to decrease the incidence of meningitis in adolescents and young adults. Which nursing action is most important? a. Vaccinate 11- and 12-year-old children against Haemophilus influenzae. b. Emphasize the importance of hand washing to prevent spread of infection. c. Immunize adolescents and college freshman against Neisseria meningitides. d. Encourage adolescents and young adults to avoid crowded areas in the winter.

ANS: C The Neisseria meningitides vaccination is recommended for children ages 11 and 12, unvaccinated teens entering high school, and college freshmen. Hand washing may help decrease the spread of bacteria, but it is not as effective as immunization. Vaccination with Haemophilus influenzae is for infants and toddlers. Because adolescents and young adults are in school or the workplace, avoiding crowds is not realistic. DIF: Cognitive Level: Application REF: 1453-1455

Which information about a patient who is hospitalized after a traumatic brain injury requires the most rapid action by the nurse? a. Intracranial pressure of 15 mm Hg b. Cerebrospinal fluid (CSF) drainage of 15 mL/hour c. Pressure of oxygen in brain tissue (PbtO2) is 14 mm Hg d. Cardiac monitor shows sinus tachycardia, with a heart rate of 126 beats/min

ANS: C The PbtO2 should be 20 to 40 mm Hg. Lower levels indicate brain ischemia. An intracranial pressure (ICP) of 15 mm Hg is at the upper limit of normal. CSF is produced at a rate of 20 to 30 mL/hour. The reason for the sinus tachycardia should be investigated, but the elevated heart rate is not as concerning as the decrease in PbtO2. DIF: Cognitive Level: Application REF: 1430-1432

A 22-year-old patient seen at the health clinic with a severe migraine headache tells the nurse about having other similar headaches recently. Which initial action should the nurse take? a. Teach about the use of triptan drugs. b. Refer the patient for stress counseling. c. Ask the patient to keep a headache diary. d. Suggest the use of muscle-relaxation techniques.

ANS: C The initial nursing action should be further assessment of the precipitating causes of the headaches, quality, and location of pain, etc. Stress reduction, muscle relaxation, and the triptan drugs may be helpful, but more assessment is needed first. DIF: Cognitive Level: Apply (application) REF: 1419 OBJ: Special Questions: Prioritization TOP: Nursing Process: Implementation MSC: NCLEX: Physiological Integrity

A patient has a systemic BP of 108/51 mm Hg and an intracranial pressure (ICP) of 14 mm Hg. Which action should the nurse take first? a. Elevate the head of the patient's bed to 60 degrees. b. Document the BP and ICP in the patient's record. c. Report the BP and ICP to the health care provider. d. Continue to monitor the patient's vital signs and ICP.

ANS: C The patient's cerebral perfusion pressure is 56 mm Hg, below the normal of 60 to 100 mm Hg and approaching the level of ischemia and neuronal death. Immediate changes in the patient's therapy such as fluid infusion or vasopressor administration are needed to improve the cerebral perfusion pressure. Adjustments in the head elevation should only be done after consulting with the health care provider. Continued monitoring and documentation also will be done, but they are not the first actions that the nurse should take. DIF: Cognitive Level: Analysis REF: 1426

A 76-year-old patient is being treated with carbidopa/levodopa (Sinemet) for Parkinson's disease. Which information is most important for the nurse to report to the health care provider? a. Shuffling gait b. Tremor at rest c. Cogwheel rigidity of limbs d. Uncontrolled head movement

ANS: D Dyskinesia is an adverse effect of the Sinemet, indicating a need for a change in medication or decrease in dose. The other findings are typical with Parkinson's disease. DIF: Cognitive Level: Apply (application) REF: 1435 OBJ: Special Questions: Prioritization TOP: Nursing Process: Planning MSC: NCLEX: Physiological Integrity

The care plan for a patient who has increased intracranial pressure and a ventriculostomy includes the following nursing actions. Which action can the nurse delegate to nursing assistive personnel (NAP) who regularly work in the intensive care unit? a. Monitor cerebrospinal fluid color hourly. b. Document intracranial pressure every hour. c. Turn and reposition the patient every 2 hours. d. Check capillary blood glucose level every 6 hours.

ANS: D Experienced NAP can obtain capillary blood glucose levels when they have been trained and evaluated in the skill. Monitoring and documentation of cerebrospinal fluid (CSF) color and intracranial pressure (ICP) require RN-level education and scope of practice. Although repositioning patients is frequently delegated to NAP, repositioning a patient with a ventriculostomy is complex and should be done by the RN. DIF: Cognitive Level: Application REF: 1442

During the neurologic assessment, the patient cooperates with the nurse's directions to grip with the hands and to move the feet but is unable to respond orally to the nurse's questions. The nurse will suspect a. a brainstem lesion. b. a temporal lobe lesion. c. injury to the cerebellum. d. damage to the frontal lobe.

ANS: D Expressive speech is controlled by Broca's area in the frontal lobe. The temporal lobe contains Wernicke's area, which is responsible for receptive speech. The cerebellum and brainstem do not affect higher cognitive functions such as speech. DIF: Cognitive Level: Application REF: 1408-1409

A 62-year-old patient who has Parkinson's disease is taking bromocriptine (Parlodel). Which information obtained by the nurse may indicate a need for a decrease in the dose? a. The patient has a chronic dry cough. b. The patient has four loose stools in a day. c. The patient develops a deep vein thrombosis. d. The patient's blood pressure is 92/52 mm Hg.

ANS: D Hypotension is an adverse effect of bromocriptine, and the nurse should check with the health care provider before giving the medication. Diarrhea, cough, and deep vein thrombosis are not associated with bromocriptine use. DIF: Cognitive Level: Apply (application) REF: 1435 TOP: Nursing Process: Evaluation MSC: NCLEX: Physiological Integrity

Which parameter is best for the nurse to monitor to determine whether the prescribed IV mannitol (Osmitrol) has been effective for an unconscious patient? a. Hematocrit b. Blood pressure c. Oxygen saturation d. Intracranial pressure

ANS: D Mannitol is an osmotic diuretic and will reduce cerebral edema and intracranial pressure. It may initially reduce hematocrit and increase blood pressure, but these are not the best parameters for evaluation of the effectiveness of the drug. Oxygen saturation will not directly improve as a result of mannitol administration. DIF: Cognitive Level: Application REF: 1432-1433

Which information about a 72-year-old patient who has a new prescription for phenytoin (Dilantin) indicates that the nurse should consult with the health care provider before administration of the medication? a. Patient has generalized tonic-clonic seizures. b. Patient experiences an aura before seizures. c. Patient's most recent blood pressure is 156/92 mm Hg. d. Patient has minor elevations in the liver function tests.

ANS: D Many older patients (especially with compromised liver function) may not be able to metabolize phenytoin. The health care provider may need to choose another antiseizure medication. Phenytoin is an appropriate medication for patients with tonic-clonic seizures, with or without an aura. Hypertension is not a contraindication for phenytoin therapy. DIF: Cognitive Level: Apply (application) REF: 1424 TOP: Nursing Process: Implementation MSC: NCLEX: Physiological Integrity

Which assessment finding in a patient who was admitted the previous day with a basilar skull fracture is most important to report to the health care provider? a. Bruising under both eyes b. Complaint of severe headache c. Large ecchymosis behind one ear d. Temperature of 101.5° F (38.6° C)

ANS: D Patients who have basilar skull fractures are at risk for meningitis, so the elevated temperature should be reported to the health care provider. The other findings are typical of a patient with a basilar skull fracture. DIF: Cognitive Level: Application REF: 1440

After having a craniectomy and left anterior fossae incision, a patient has a nursing diagnosis of impaired physical mobility related to decreased level of consciousness and weakness. An appropriate nursing intervention is to a. position the bed flat and log roll the patient. b. cluster nursing activities to allow longer rest periods. c. turn and reposition the patient side to side every 2 hours. d. perform range-of-motion (ROM) exercises every 4 hours.

ANS: D ROM exercises will help to prevent the complications of immobility. Patients with anterior craniotomies are positioned with the head elevated. The patient with a craniectomy should not be turned to the operative side. When the patient is weak, clustering nursing activities may lead to more fatigue and weakness. DIF: Cognitive Level: Application REF: 1450-1451

A patient is hospitalized with a possible seizure disorder. To determine the cause of the patient's symptoms, the nurse will anticipate the need to teach the patient about which of these tests? a. Cerebral angiography b. Evoked potential studies c. Electromyography (EMG) d. Electroencephalography (EEG)

ANS: D Seizure disorders are usually studied using EEG testing. Evoked potential is used for diagnosing problems with the visual or auditory systems. Cerebral angiography is used to diagnose vascular problems. EMG is used to evaluate electrical innervation to skeletal muscle. DIF: Cognitive Level: Comprehension REF: 1419-1423

When assessing a patient with bacterial meningitis, the nurse obtains the following data. Which finding should be reported immediately to the health care provider? a. The patient has a positive Kernig's sign. b. The patient complains of having a stiff neck. c. The patient's temperature is 101° F (38.3° C). d. The patient's blood pressure is 86/42 mm Hg.

ANS: D Shock is a serious complication of meningitis, and the patient's low blood pressure indicates the need for interventions such as fluids or vasopressors. Nuchal rigidity and a positive Kernig's sign are expected with bacterial meningitis. The nurse should intervene to lower the temperature, but this is not as life threatening as the hypotension. DIF: Cognitive Level: Application REF: 1452-1453

hich nursing diagnosis is of highest priority for a patient with Parkinson's disease who is unable to move the facial muscles? a. Activity intolerance b. Self-care deficit: toileting c. Ineffective self-health management d. Imbalanced nutrition: less than body requirements

ANS: D The data about the patient indicate that poor nutrition will be a concern because of decreased swallowing. The other diagnoses may also be appropriate for a patient with Parkinson's disease, but the data do not indicate that they are current problems for this patient. DIF: Cognitive Level: Apply (application) REF: 1436 OBJ: Special Questions: Prioritization TOP: Nursing Process: Analysis MSC: NCLEX: Physiological Integrity

After the emergency department nurse has received a status report on the following patients who have been admitted with head injuries, which patient should the nurse assess first? a. A patient whose cranial x-ray shows a linear skull fracture b. A patient who has an initial Glasgow Coma Scale score of 13 c. A patient who lost consciousness for a few seconds after a fall d. A patient whose right pupil is 10 mm and unresponsive to light

ANS: D The dilated and nonresponsive pupil may indicate an intracerebral hemorrhage and increased intracranial pressure. The other patients are not at immediate risk for complications such as herniation. DIF: Cognitive Level: Analysis REF: 1432-1433 | 1437-1438

When performing a focused assessment on a patient with a lesion of the left posterior temporal lobe, the nurse will assess for a. sensation on the left side of the body. b. voluntary movement on the right side. c. reasoning and problem-solving abilities. d. understanding of written and oral language.

ANS: D The posterior temporal lobe integrates the visual and auditory input for language comprehension. Reasoning and problem solving are functions of the anterior frontal lobe. Sensation on the left side of the body is located in the right postcentral gyrus. Voluntary movement on the right side is controlled in the left precentral gyrus. DIF: Cognitive Level: Application REF: 1409

A patient who is suspected of having an epidural hematoma is admitted to the emergency department. Which action will the nurse plan to take? a. Administer IV furosemide (Lasix). b. Initiate high-dose barbiturate therapy. c. Type and crossmatch for blood transfusion. d. Prepare the patient for immediate craniotomy.

ANS: D The principal treatment for epidural hematoma is rapid surgery to remove the hematoma and prevent herniation. If intracranial pressure (ICP) is elevated after surgery, furosemide or high-dose barbiturate therapy may be needed, but these will not be of benefit unless the hematoma is removed. Minimal blood loss occurs with head injuries, and transfusion is usually not necessary. DIF: Cognitive Level: Application REF: 1440-1441

Which prescribed intervention will the nurse implement first for a patient in the emergency department who is experiencing continuous tonic-clonic seizures? a. Give phenytoin (Dilantin) 100 mg IV. b. Monitor level of consciousness (LOC). c. Obtain computed tomography (CT) scan. d. Administer lorazepam (Ativan) 4 mg IV.

ANS: D To prevent ongoing seizures, the nurse should administer rapidly acting antiseizure medications such as the benzodiazepines. A CT scan is appropriate, but prevention of any seizure activity during the CT scan is necessary. Phenytoin will also be administered, but it is not rapidly acting. Patients who are experiencing tonic-clonic seizures are nonresponsive, although the nurse should assess LOC after the seizure. DIF: Cognitive Level: Apply (application) REF: 1424 OBJ: Special Questions: Prioritization TOP: Nursing Process: Implementation MSC: NCLEX: Physiological Integrity

A patient with a brainstem infarction is admitted to the nursing unit. The priority nursing assessment for the patient is a. reflex reaction time. b. pupil reaction to light. c. level of consciousness. d. respiratory rate and rhythm.

ANS: D Vital centers that control respiration are located in the medulla, and these are the priority assessments because changes in respiratory function may be life threatening. The other information also will be collected by the nurse, but it is not as urgent. DIF: Cognitive Level: Application REF: 1410

The nurse is caring for a patient admitted with a subdural hematoma following a motor vehicle accident. Which change in vital signs would the nurse interpret as a manifestation of increased intracranial pressure (ICP)? A. Tachypnea B. Bradycardia C. Hypotension D. Narrowing pulse pressure

B Bradycardia could indicate increased ICP. Changes in vital signs (known as Cushing's triad) occur with increased ICP. They consist of increasing systolic pressure with a widening pulse pressure, bradycardia with a full and bounding pulse, and irregular respirations.

The nurse is caring for a patient admitted with a subdural hematoma following a motor vehicle accident. Which change in vital signs would the nurse interpret as a manifestation of increased intracranial pressure (ICP)? A. Tachypnea B. Bradycardia C. Hypotension D. Narrowing pulse pressure

B Bradycardia could indicate increased ICP. Changes in vital signs (known as Cushing's triad) occur with increased ICP. They consist of increasing systolic pressure with a widening pulse pressure, bradycardia with a full and bounding pulse, and irregular respirations.

Decerebrate posture is documented in the chart of the patient that the nurse will be caring for. The nurse should know that the patient may have elevated ICP causing serious disruption of motor fibers in the midbrain and brainstem and will expect the patient's posture to look like which posture represented below?

B Decerebrate posture is all four extremities in rigid extension with hyperpronation of the forearms and plantar flexion of feet. Decorticate posture is internal rotation and adduction of the arms with flexion of the elbows, wrists, and fingers from interruption of voluntary motor tracts in the cerebral cortex. Decorticate response on one side of the body and decerebrate response on the other side of the body may occur depending on the damage to the brain. Opisthotonic posture is decerebrate posture with the neck and back arched posteriorly and may be seen with traumatic brain injury.

A 32-year-old female patient is diagnosed with diabetes insipidus after transsphenoidal resection of a pituitary adenoma. What should the nurse consider as a sign of improvement? A. Serum sodium of 120 mEq/L B. Urine specific gravity of 1.001 C. Fasting blood glucose of 80 mg/dL D. Serum osmolality of 290 mOsm/kg

B Laboratory findings in diabetes insipidus include an elevation in serum osmolality and serum sodium and a decrease in urine specific gravity. Normal serum osmolality is 275 to 295 mOsm/kg, normal serum sodium is 135 to 145 mEq/L, and normal specific gravity is 1.003 to 1.030. Elevated blood glucose levels occur with diabetes mellitus.

The nurse provides information to the caregiver of a 68-year-old man with epilepsy who has tonic-clonic seizures. Which statement, if made by the caregiver, requires further teaching? A. "It is normal for a person to be sleepy after a seizure." B. "I should call 911 if breathing stops during the seizure." C. "The jerking movements may last for 30 to 40 seconds." D. "Objects should not be placed in the mouth during a seizure."

B. Caregivers do not need to call an ambulance or send a person to the hospital after a single seizure unless the seizure is prolonged, another seizure immediately follows, or extensive injury has occurred. Altered breathing is a clinical manifestation of a tonic-clonic seizure. Contact emergency medical services (or call 911) if breathing stops for more than 30 seconds. No objects (e.g., oral airway, padded tongue blade) should be placed in the mouth. Lethargy is common in the postictal phase of a seizure. Jerking of the extremities occurs during the clonic phase of a tonic-clonic seizure. The clonic phase may last 30 to 40 seconds.

A patient has been receiving scheduled doses of phenytoin (Dilantin) and begins to experience diplopia. The nurse immediately assesses the patient for A. an aura or focal seizure. B. nystagmus or confusion. C. abdominal pain or cramping. D. irregular pulse or palpitations.

B. Diplopia is a sign of phenytoin toxicity. The nurse should assess for other signs of toxicity, which include neurologic changes, such as nystagmus, ataxia, confusion, dizziness, or slurred speech. An aura, focal seizure, abdominal pain or cramping, irregular pulse, or palpitations are not associated with phenytoin toxicity.

Classic symptoms of bacterial meningitis include a. papilledema and psychomotor seizures b. high fever, nuchal rigidity, and severe headache c. behavioral changes with memory loss and lethargy d. positive Kernig's and Brudzinski's signs and hemiparesis

B. High fever, severe headache, nuchal rigidity, and positive Brudzinski's and Kernig's signs are such classic symptoms of meningitis that they are usually considered diagnostic for meningitis. Other symptoms, such as papilledema, generalized seizures, hemiparesis, and decreased LOC, may occur as complications of increased ICP and cranial nerve dysfunction.

The nurse observes a 74-year-old man with Parkinson's disease rocking side to side while sitting in the chair. Which action by the nurse is most appropriate? A. Provide the patient with diversional activities. B. Document the activity in the patient's health record. C. Take the patient's blood pressure sitting and standing. D. Ask if the patient is feeling either anxious or depressed.

B. Patients with Parkinson's disease are instructed to rock from side to side to stimulate balance mechanisms and decrease akinesia.

A 50-year-old male patient has been diagnosed with amyotrophic lateral sclerosis (ALS). What nursing intervention is most important to help prevent a common cause of death for patients with ALS? A. Reduce fat intake. B. Reduce the risk of aspiration. C. Decrease injury related to falls. D. Decrease pain secondary to muscle weakness.

B. Reducing the risk of aspiration can help prevent respiratory infections that are a common cause of death from deteriorating muscle function. Reducing fat intake may reduce cardiovascular disease, but this is not a common cause of death for patients with ALS. Decreasing injury related to falls and decreasing pain secondary to muscle weakness are important nursing interventions for patients with ALS but are unrelated to causes of death for these patients.

A patient with an intracranial problem does not open his eyes to any stimulus, has no verbal response except moaning and muttering when stimulated, and flexes his arm in response to painful stimuli. The nurse records the patients GCS score as a. 6 b. 7 c. 9 d. 11

B. no opening of eyes = 1 incomprehensible words= 2 flexion withdrawal = 4 Total = 7

The nurse recognizes the presence of Cushing's triad in the patient with a. Increased pulse, irregular respiration, increased BP b. decreased pulse, irregular respiration, increased pulse pressure c. increased pulse, decreased respiration, increased pulse pressure d. decreased pulse, increased respiration, decreased systolic BP

B. Cushing's triad consists of three vital sign measures that reflect ICP and its effect on the medulla, the hypothalamus, the pons, and the thalamus. Because these structures are very deep, Cushing's triad is usually a late sign of ICP. The signs include an increasing systolic BP with a widening pulse pressure, a bradycardia with a full and bounding pulse, and irregular respirations.

Assisting the family to understand what is happening to the patient is an especially important role of the nurse when the patient has a tumor of the a. ventricles b. frontal lobe c. parietal lobe d. occipital lobe

B. Frontal lobe tumors often lead to loss of emotional control, confusion, memory loss, disorientation, and personality changes that are very disturbing and frightening to the family. Physical symptoms, such as blindness, disturbances in sensation and perception, and even seizures, that occur with other tumors are more likely to be understood and accepted by the family

A 73-year-old patient with Parkinson's disease has a nursing diagnosis of impaired physical mobility related to bradykinesia. Which action will the nurse include in the plan of care? a. Instruct the patient in activities that can be done while lying or sitting. b. Suggest that the patient rock from side to side to initiate leg movement. c. Have the patient take small steps in a straight line directly in front of the feet. d. Teach the patient to keep the feet in contact with the floor and slide them forward.

B. Rocking the body from side to side stimulates balance and improves mobility. The patient will be encouraged to continue exercising because this will maintain functional abilities. Maintaining a wide base of support will help with balance. The patient should lift the feet and avoid a shuffling gait.

The nurse teaches a 38-year-old woman who has migraine headaches about sumatriptan (Imitrex). Which statement by the patient requires an intervention by the nurse? A. "The injection might feel like a bee sting." B. "This medicine will prevent a migraine headache." C. "I can take another dose if the first does not work." D. "This drug for migraine headaches could cause birth defects."

B. Sumatriptan is given to abort an ongoing migraine headache and is not used to prevent migraine headaches. When given as a subcutaneous injection, this drug may cause transient pain and redness at the injection site. This drug may be repeated after a specified time period if the first dose is not effective. This drug should be avoided during pregnancy and is classified as an FDA Pregnancy Risk Category C drug.

A patient with a head injury has bloody drainage from the ear. To determine whether CSF is present in the drainage, the nurse a. examines the tympanic membrane for a tear b. tests the fluid for a halo sign on a white dressing c. tests the fluid with a glucose identifying strip or stick d. collects 5 mL of fluid in a test tube and sends it to the laboratory for analysis

B. Testing clear drainage for CSF in nasal or ear drainage may be done with a Dextrostik or Tes-Tape strip, but if blood is present, the glucose in the blood will produce and unreliable result. To test bloody drainage, the nurse should test the fluid for a halo or ring that occurs when a yellowish ring encircles blood dripped onto a white pad or towel

Which action will the nurse plan to take for a 40-year-old patient with multiple sclerosis (MS) who has urinary retention caused by a flaccid bladder? a. Decrease the patient's evening fluid intake. b. Teach the patient how to use the Credé method. c. Suggest the use of adult incontinence briefs for nighttime only. d. Assist the patient to the commode every 2 hours during the day.

B. The Credé method can be used to improve bladder emptying. Decreasing fluid intake will not improve bladder emptying and may increase risk for urinary tract infection (UTI) and dehydration. The use of incontinence briefs and frequent toileting will not improve bladder emptying.

When a 74-year-old patient is seen in the health clinic with new development of a stooped posture, shuffling gait, and pill rolling-type tremor, the nurse will anticipate teaching the patient about a. oral corticosteroids. b. antiparkinsonian drugs. c. magnetic resonance imaging (MRI). d. electroencephalogram (EEG) testing.

B. The diagnosis of Parkinson's is made when two of the three characteristic manifestations of tremor, rigidity, and bradykinesia are present. The confirmation of the diagnosis is made on the basis of improvement when antiparkinsonian drugs are administered. This patient has symptoms of tremor and bradykinesia. The next anticipated step will be treatment with medications. MRI and EEG are not useful in diagnosing Parkinson's disease, and corticosteroid therapy is not used to treat it.

Which characteristic of a patient's recent seizure is consistent with a focal seizure? A. The patient lost consciousness during the seizure. B. The seizure involved lip smacking and repetitive movements. C. The patient fell to the ground and became stiff for 20 seconds. D. The etiology of the seizure involved both sides of the patient's brain.

B. The most common complex focal seizure involves lip smacking and automatisms (repetitive movements that may not be appropriate). Loss of consciousness, bilateral brain involvement, and a tonic phase are associated with generalized seizure activity.

Following a thymectomy, a 62-year-old male patient with myasthenia gravis receives the usual dose of pyridostigmine (Mestinon). An hour later, the patient complains of nausea and severe abdominal cramps. Which action should the nurse take first? a. Auscultate the patient's bowel sounds. b. Notify the patient's health care provider. c. Administer the prescribed PRN antiemetic drug. d. Give the scheduled dose of prednisone (Deltasone).

B. The patient's history and symptoms indicate a possible cholinergic crisis. The health care provider should be notified immediately, and it is likely that atropine will be prescribed. The other actions will be appropriate if the patient is not experiencing a cholinergic crisis.

When obtaining a health history and physical assessment for a 36-year-old female patient with possible multiple sclerosis (MS), the nurse should a. assess for the presence of chest pain. b. inquire about urinary tract problems. c. inspect the skin for rashes or discoloration. d. ask the patient about any increase in libido.

B. Urinary tract problems with incontinence or retention are common symptoms of MS. Chest pain and skin rashes are not symptoms of MS. A decrease in libido is common with MS.

When assessing the accessory nerve, what should the nurse do? A. Assess the gag reflex by stroking the posterior pharynx. B. Ask the patient to shrug the shoulders against resistance. C. Ask the patient to push the tongue to either side against resistance. D. Have the patient say "ah" while visualizing elevation of soft palate.

B. Ask the patient to shrug the shoulders against resistance. The spinal accessory nerve is tested by asking the patient to shrug the shoulders against resistance and to turn the head to either side against resistance while observing the sternocleidomastoid muscles and the trapezius muscles. Assessing the gag reflex and saying "ah" are used to assess the glossopharyngeal and vagus nerves. Asking the patient to push the tongue to either side against resistance and to stick out the tongue are used to assess the hypoglossal nerve.

How should the nurse most accurately assess the position sense of a patient with a recent traumatic brain injury? A. Ask the patient to close his or her eyes and slowly bring the tips of the index fingers together. B. Ask the patient to stand with the feet together and eyes closed and observe for balance maintainance. C. Ask the patient to close his or her eyes and identify the presence of a common object on the forearm. D. Place the two points of a calibrated compass on the tips of the fingers and toes and ask the patient to discriminate the points.

B. Ask the patient to stand with the feet together and eyes closed and observe for balance maintainance. The Romberg test is an assessment of position sense in which the patient stands with the feet together and then closes his or her eyes while attempting to maintain balance. The other cited tests of neurologic function do not directly assess position sense.

A 68-year-old man with suspected bacterial meningitis has just had a lumbar puncture in which cerebrospinal fluid was obtained for culture. Which medication should the nurse administer first? A. Codeine B. Phenytoin (Dilantin) C. Ceftriaxone (Rocephin) D. Acetaminophen (Tylenol)

C Bacterial meningitis is a medical emergency. When meningitis is suspected, antibiotic therapy (e.g., ceftriaxone) is instituted immediately after the collection of specimens for cultures, and even before the diagnosis is confirmed. Dexamethasone may also be prescribed before or with the first dose of antibiotics. The nurse should collaborate with the health care provider to manage the headache (with codeine), fever (with acetaminophen), and seizures (with phenytoin).

In planning long-term care for a patient after a craniotomy, what must the nurse include when teaching the patient, family, and caregiver? A. Seizure disorders may occur in weeks or months. B. The family will be unable to cope with role reversals. C. There are often residual changes in personality and cognition. D. Referrals will be made to eliminate residual deficits from the damage.

C In long-term care planning, the nurse must include the family and caregiver when teaching about potential residual changes in personality, emotions, and cognition as these changes are most difficult for the patient and family to accept. Seizures may or may not develop. The family and patient may or may not be able to cope with role reversals. Although residual deficits will not be eliminated with referrals, they may be improved.

In planning long-term care for a patient after a craniotomy, what must the nurse include when teaching the patient, family, and caregiver? A. Seizure disorders may occur in weeks or months. B. The family will be unable to cope with role reversals. C. There are often residual changes in personality and cognition. D. Referrals will be made to eliminate residual deficits from the damage.

C In long-term care planning, the nurse must include the family and caregiver when teaching about potential residual changes in personality, emotions, and cognition as these changes are most difficult for the patient and family to accept. Seizures may or may not develop. The family and patient may or may not be able to cope with role reversals. Although residual deficits will not be eliminated with referrals, they may be improved.

A patient has a systemic blood pressure of 120/60 and an ICP of 24 mm Hg. After calculating the patient's cerebral perfusion pressure (CPP), how does the nurse interpret the results? A. High blood flow to the brain B. Normal intracranial pressure C. Impaired blood flow to the brain D. Adequate autoregulation of blood flow

C Normal CPP is 60 to 100 mm Hg. The CPP is calculated with mean arterial pressure (MAP) minus ICP. MAP = SBP + 2 (DBP)/ 3: 120 mm Hg + 2 (60 mm Hg)/3 = 80 mm Hg. MAP - ICP: 80mm Hg - 24 mm Hg = 56 mm Hg CPP. The decreased CPP indicates that there is impaired cerebral blood flow and that autoregulation is impaired. Because the ICP is 24, it is elevated and requires treatment.

A patient has a systemic blood pressure of 120/60 and an ICP of 24 mm Hg. After calculating the patient's cerebral perfusion pressure (CPP), how does the nurse interpret the results? A. High blood flow to the brain B. Normal intracranial pressure C. Impaired blood flow to the brain D. Adequate autoregulation of blood flow

C Normal CPP is 60 to 100 mm Hg. The CPP is calculated with mean arterial pressure (MAP) minus ICP. MAP = SBP + 2 (DBP)/ 3: 120 mm Hg + 2 (60 mm Hg)/3 = 80 mm Hg. MAP - ICP: 80mm Hg - 24 mm Hg = 56 mm Hg CPP. The decreased CPP indicates that there is impaired cerebral blood flow and that autoregulation is impaired. Because the ICP is 24, it is elevated and requires treatment.

The nurse prepares to administer temozolomide (Temodar) to a 59-year-old white male patient with a glioblastoma multiforme (GBM) brain tumor. What should the nurse assess before giving the medication? A. Serum potassium and serum sodium levels B. Urine osmolality and urine specific gravity C. Absolute neutrophil count and platelet count D. Cerebrosprinal fluid (CSF) pressure and cell count

C Temozolomide causes myelosuppression. The nurse should assess the absolute neutrophil count and the platelet count. The absolute neutrophil count should be >1500/μL and platelet count >100,000/μL.

The nurse prepares to administer temozolomide (Temodar) to a 59-year-old white male patient with a glioblastoma multiforme (GBM) brain tumor. What should the nurse assess before giving the medication? A. Serum potassium and serum sodium levels B. Urine osmolality and urine specific gravity C. Absolute neutrophil count and platelet count D. Cerebrosprinal fluid (CSF) pressure and cell count

C Temozolomide causes myelosuppression. The nurse should assess the absolute neutrophil count and the platelet count. The absolute neutrophil count should be >1500/μL and platelet count >100,000/μL.

The physician orders intracranial pressure (ICP) readings every hour for a 23-year-old male patient with a traumatic brain injury from a motor vehicle crash. The patient's ICP reading is 21 mm Hg. It is most important for the nurse to take which action? A. Document the ICP reading in the chart. B. Determine if the patient has a headache. C. Assess the patient's level of consciousness. D. Position the patient with head elevated 60 degrees.

C The patient has an increased ICP (normal ICP ranges from 5 to 15 mm Hg). The most sensitive and reliable indicator of neurologic status is level of consciousness. The Glasgow Coma Scale may be used to determine the degree of impaired consciousness.

The physician orders intracranial pressure (ICP) readings every hour for a 23-year-old male patient with a traumatic brain injury from a motor vehicle crash. The patient's ICP reading is 21 mm Hg. It is most important for the nurse to take which action? A. Document the ICP reading in the chart. B. Determine if the patient has a headache. C. Assess the patient's level of consciousness. D. Position the patient with head elevated 60 degrees.

C The patient has an increased ICP (normal ICP ranges from 5 to 15 mm Hg). The most sensitive and reliable indicator of neurologic status is level of consciousness. The Glasgow Coma Scale may be used to determine the degree of impaired consciousness.

The patient with increased ICP from a brain tumor is being monitored with a ventriculostomy. What nursing intervention is the priority in caring for this patient? A. Administer IV mannitol (Osmitrol). B. Ventilator use to hyperoxygenate the patient C. Use strict aseptic technique with dressing changes. D. Be aware of changes in ICP related to leaking CSF.

C The priority nursing intervention is to use strict aseptic technique with dressing changes and any handling of the insertion site to prevent the serious complication of infection. IV mannitol (Osmitrol) or hypertonic saline will be administered as ordered. Ventilators may be used to maintain oxygenation. CSF leaks may cause inaccurate ICP readings, or CSF may be drained to decrease ICP, but strict aseptic technique to prevent infection is the nurse's priority of care.

The patient with increased ICP from a brain tumor is being monitored with a ventriculostomy. What nursing intervention is the priority in caring for this patient? A. Administer IV mannitol (Osmitrol). B. Ventilator use to hyperoxygenate the patient C. Use strict aseptic technique with dressing changes. D. Be aware of changes in ICP related to leaking CSF.

C The priority nursing intervention is to use strict aseptic technique with dressing changes and any handling of the insertion site to prevent the serious complication of infection. IV mannitol (Osmitrol) or hypertonic saline will be administered as ordered. Ventilators may be used to maintain oxygenation. CSF leaks may cause inaccurate ICP readings, or CSF may be drained to decrease ICP, but strict aseptic technique to prevent infection is the nurse's priority of care.

A female patient complains of a throbbing headache. When her history is obtained, the nurse discovers that the patient has had this type of headache before and experienced photophobia before the headache occurred. The nurse should know that what is probably the cause of this patient's headache? A. Polycythemia vera B. A cluster headache C. A migraine headache D. A hemorrhagic stroke

C. Although a headache may occur with any of these options, a migraine headache is the only one that has a throbbing headache with an aura (the photophobia). Headache from polycythemia vera is from erythrocytosis. The cluster headache pain is sharp and stabbing, and the headache with a hemorrhagic stroke has a sudden onset and is not recurrent.

A male patient with a diagnosis of Parkinson's disease (PD) has been admitted recently to a long-term care facility. Which action should the health care team take in order to promote adequate nutrition for this patient? A. Provide multivitamins with each meal. B. Provide a diet that is low in complex carbohydrates and high in protein. C. Provide small, frequent meals throughout the day that are easy to chew and swallow. D. Provide the patient with a minced or pureed diet that is high in potassium and low in sodium.

C. Nutritional support is a priority in the care of individuals with PD. Such patients may benefit from meals that are smaller and more frequent than normal and that are easy to chew and swallow. Multivitamins are not necessary at each meal, and vitamin intake, along with protein intake, must be monitored to prevent contraindications with medications. It is likely premature to introduce a minced or pureed diet, and a low carbohydrate diet is not indicated.

The nurse provides dietary instructions to the in-home caregiver of a 45-year-old man who has Huntington's disease. The nurse is most concerned if the caregiver makes which statement? A. "Depression is common and may cause a decrease in appetite." B. "If swallowing becomes difficult, a feeding tube may be needed." C. "Calories should be restricted to prevent unnecessary weight gain." D. "Muscles in the face are affected, and chewing may become impossible."

C. Patients with Huntington's disease may require 4000 to 5000 calories per day to maintain body weight. Weight loss occurs in patients with Huntington's disease because of choreic movements, difficulty swallowing, depression, and mental deterioration.

Which nursing diagnosis is likely to be a priority in the care of a patient with myasthenia gravis (MG)? A. Acute confusion B. Bowel incontinence C. Activity intolerance D. Disturbed sleep pattern

C. The primary feature of MG is fluctuating weakness of skeletal muscle. Bowel incontinence and confusion are unlikely signs of MG, and although sleep disturbance is likely, activity intolerance is usually of primary concern.

The nurse is caring for a group of patients on a medical unit. After receiving report, which patient should the nurse see first? A. A 42-year-old patient with multiple sclerosis who was admitted with sepsis B. A 72-year-old patient with Parkinson's disease who has aspiration pneumonia C. A 38-year-old patient with myasthenia gravis who declined prescribed medications D. A 45-year-old patient with amyotrophic lateral sclerosis who refuses enteral feedings

C. Patients with myasthenia gravis who discontinue pyridostigmine (Mestinon) will develop a myasthenic crisis. Myasthenia crisis results in severe muscle weakness and can lead to a respiratory arrest.

An unconscious patient with increased ICP is on ventilatory support. The nurse notifies the health care provider when arterial blood gas measurement results reveal a a. pH of 7.43 b. SaO2 of 94% c. PaO2 of 50 mm Hg d. PaCO2 of 30 mm Hg

C. A PaO2 of 50 mm Hg reflects a hypoxemia that may lead to further decreased cerebral perfusion and hypoxia and must be corrected. The pH of SaO2 are within normal range, and a PaCO2 of 30 mm Hg reflects acceptable value for the patient with increased ICP

Which assessment is most important for the nurse to make regarding a patient with myasthenia gravis? a. Pupil size b. Grip strength c. Respiratory effort d. Level of consciousness

C. Because respiratory insufficiency may be life threatening, it will be most important to monitor respiratory function. The other data also will be assessed but are not as critical.

Increased ICP in the left cerebral cortex, caused by intracranial bleeding causes displacement of brain tissue to the right hemisphere beneath the falx cerebri. The nurse knows that this is referred to as a. uncal herniation b. tentorial herniation c. cingulate herniation d. temporal lobe herniation

C. Cingulate herniation- the dural structures that separate the two hemispheres and the cerebral hemispheres from the cerebellum influence the patterns of cerebral herniation. A cingulated herniation occurs where there is lateral displacement of brain tissue beneath the falx cerebri.

A 49-year-old patient with multiple sclerosis (MS) is to begin treatment with glatiramer acetate (Copaxone). Which information will the nurse include in patient teaching? a. Recommendation to drink at least 4 L of fluid daily b. Need to avoid driving or operating heavy machinery c. How to draw up and administer injections of the medication d. Use of contraceptive methods other than oral contraceptives

C. Copaxone is administered by self-injection. Oral contraceptives are an appropriate choice for birth control. There is no need to avoid driving or drink large fluid volumes when taking glatiramer.

A 40-year-old patient is diagnosed with early Huntington's disease (HD). When teaching the patient, spouse, and children about this disorder, the nurse will provide information about the a. use of levodopa-carbidopa (Sinemet) to help reduce HD symptoms. b. prophylactic antibiotics to decrease the risk for aspiration pneumonia. c. option of genetic testing for the patient's children to determine their own HD risks. d. lifestyle changes of improved nutrition and exercise that delay disease progression.

C. Genetic testing is available to determine whether an asymptomatic individual has the HD gene. The patient and family should be informed of the benefits and problems associated with genetic testing. Sinemet will increase symptoms of HD because HD involves an increase in dopamine. Antibiotic therapy will not reduce the risk for aspiration. There are no effective treatments or lifestyle changes that delay the progression of symptoms in HD.

While the nurse performs ROM on an unconscious patient with increased ICP, the patient experiences severe decerebrate posturing reflexes. The nurse should a. use restraints to protect the patient from injury b. administer CNS depressants to lightly sedate the patient c. perform the exercises less frequently because posturing can increase ICP d. continue the exercises because they are necessary to maintain musculoskeletal function

C. If reflex posturing occurs during ROM or positioning of the patient, these activities should be done less frequently until the patient's condition stabilizes, because posturing can case increases in ICP. Neither restraints nor CNS depressants would be indicated.

CN III originating in the midbrain is assessed by the nurse for an early indication of pressure on the brainstem by a. assessing for nystagmus b. testing the corneal reflex c. testing pupillary reaction to light d. testing for oculocephalic (doll's eye) reflex

C. One of the functions of CN III, the oculomotor nerve, is pupillary constriction, and testing for pupillary constriction is important to identify patients at risk for brainstem herniation caused by increased ICP. The corneal reflex is used to assess the functions of CN V and VII, and the oculocephalic reflex tests all cranial nerves involved with eye movement. Nystagmus is commonly associatted with specific lesions or chemical toxicities and is not a definitive sign of ICP.

The earliest signs of increased ICP the nurse should assess for include a. Cushing's triad b. unexpected vomiting c. decreasing level of consciousness (LOC) d. dilated pupil with sluggish response to light

C. One of the most sensitive signs of increased intracranial pressure (ICP) is a decreasing LOC. A decrease in LOC will occur before changes in vital signs, ocular signs, and projectile vomiting occur

A 54-year old man is recovering from a skull fracture with a subacute subdural hematoma. He has return of motor control and orientation but appears apathetic and has reduced awareness of his environment. When planning discharge or the patient, the nurse explains to the patient and the family that a. continuous improvement in the patient's condition should occur until he has returned to pre trauma status b. the patient's complete recovery may take years, and the family should plan for his long term dependent care c. the patient is likely to have long term emotional and mental changes that may require continued professional help d. role changes in family members will be necessary because the patient will be dependent on his family for care and support

C. Residual mental and emotional changes of brain trauma with personality changes are often the most incapacitating problems following head injury and are common in patients who have been comatose longer than 6 hours. Families must be prepared for changes in the patient's behavior to avoid family-patient friction and maintain family functioning, and professional assistance may be required. There is no indication he will be dependent on others for care, but he likely will not return to pre trauma status

On physical examination of a patient with headache and fever, the nurse would suspect a brain abscess when the patient has a. seizures b. nuchal rigidity c. focal symptoms d. signs of increased ICP

C. The symptoms of brain abscess closely resemble those of meningitis and encephalitis, including fever, headache, and increased ICP, except the patient also usually has some focal symptoms that reflect the local are of the abscess.

A patient's sudden onset of hemiplegia has necessitated a computed tomography (CT) of her head. Which assessment should the nurse complete prior to this diagnostic study? A. Assess the patient's immunization history. B. Screen the patient for any metal parts or a pacemaker. C. Assess the patient for allergies to shellfish, iodine, or dyes. D. Assess the patient's need for tranquilizers or antiseizure medications.

C. Assess the patient for allergies to shellfish, iodine, or dyes. Allergies to shellfish, iodine, or dyes contraindicate the use of contrast media in CT. The patient's immunization history is not a central consideration, and the presence of metal in the body does not preclude the use of CT as a diagnostic tool. The need to assess for allergies supersedes the need for tranquilizers or antiseizure medications in the majority of patients.

The new patient has a diagnosis of frontal lobe dementia. What functional difficulties should the nurse expect in this patient? A. The lack of reflexes B. Endocrine problems C. Higher cognitive function abnormalities D. Respiratory, vasomotor, and cardiac dysfunction

C. Higher cognitive function abnormalities Because the frontal lobe is responsible for higher cognitive function, this patient may have difficulty with memory retention, voluntary eye movements, voluntary motor movement, and expressive speech. The lack of reflexes would occur if the patient had problems with the reflex arcs in the spinal cord. Endocrine problems would be evident if the hypothalamus or pituitary gland were affected. Respiratory, vasomotor, and cardiac dysfunction would occur if there were a problem in the medulla.

The nurse is preparing the patient for an electromyography (EMG). What should the nurse include in teaching the patient before the test? A. The patient will be tilted on a table during the test. B. It is noninvasive, and there is no risk of electric shock. C. The pain that occurs is from the insertion of the needles. D. The passive sensor does not make contact with the patient.

C. The pain that occurs is from the insertion of the needles. With an EMG, pain may occur when needles are inserted to record the electrical activity of nerve and skeletal muscle. The patient is tilted on a table during a myelogram. The electroencephalography (EEG) is noninvasive without a danger of electric shock. The magnetoencephalography (MEG) is done with a passive sensor that does not make contact with the patient.

18. The nurse identifies a nursing diagnosis of ineffective breathing pattern related to loss of central nervous system (CNS) integrative function for a patient who has posttraumatic brain swelling, based on the finding of a. apneustic breathing. b. crackles on inspiration. c. Glasgow Coma Scale score of 7. d. cerebral perfusion pressure of 56 mm Hg.

Correct Answer: A Rationale: Apneustic breathing is caused by loss of CNS integration in the pons and is not effective in maximizing gas exchange. Crackles on inspiration are abnormal, but they are not an indication of an abnormal breathing pattern. The Glasgow Coma Scale and cerebral perfusion pressure P are not useful in determining or documenting a patient's respiratory patterns. Cognitive Level: Application Text Reference: p. 1478 Nursing Process: Diagnosis NCLEX: Physiological Integrity

11. The charge nurse observes an inexperienced staff nurse who is caring for a patient who has had a craniotomy for a brain tumor. Which action by the inexperienced nurse requires the charge nurse to intervene? a. The staff nurse has the patient deep-breathe and cough. b. The staff nurse assesses neurologic status every hour. c. The staff nurse elevates the head of the bed to 30 degrees. d. The staff nurse administers an analgesic before turning the patient.

Correct Answer: A Rationale: Coughing can increase ICP and is generally discouraged in patients at risk for increased ICP. The other actions by the staff nurse are appropriate. Cognitive Level: Application Text Reference: pp. 1478, 1480 Nursing Process: Implementation NCLEX: Physiological Integrity

10. When a patient's intracranial pressure (ICP) is being monitored with an intraventricular catheter, which of these data obtained during the assessment is most important to communicate to the health care provider? a. Oral temperature 101.6° F b. Intracranial pressure 15 mm Hg c. Mean arterial pressure 70 mm Hg d. Apical pulse 106 beats/min

Correct Answer: A Rationale: Infection is a serious consideration with ICP monitoring, especially with intraventricular catheters; the temperature indicates the need for antibiotics or removal of the monitor. The ICP, arterial pressure, and apical pulse are all borderline high but require only ongoing monitoring at this time. Cognitive Level: Application Text Reference: p. 1473 Nursing Process: Assessment NCLEX: Physiological Integrity

9. When the nurse applies a painful stimulus to the nailbeds of an unconscious patient, the patient responds with internal rotation, adduction, and flexion of the arms. The nurse documents this as a. decorticate posturing. b. decerebrate posturing. c. localization of pain. d. flexion withdrawal.

Correct Answer: A Rationale: Internal rotation, adduction, and flexion of the arms in an unconscious patient is documented as decorticate posturing. Extension of the arms and legs is decerebrate posturing. Because the flexion is general, it does not indicate localization of pain or flexion withdrawal. Cognitive Level: Comprehension Text Reference: p. 1472 Nursing Process: Assessment NCLEX: Physiological Integrity

35. While caring for a patient who has just been admitted with meningococcal meningitis, the RN observes all of the following. Which one requires action by the RN? a. The nursing assistant goes into the patient's room without a mask. b. The bedrails at the head and foot of the bed are both elevated. c. The lights in the patient's room are turned off and the blinds are shut. d. The patient receives a regular diet from the dietary department.

Correct Answer: A Rationale: Meningococcal meningitis is spread by respiratory secretions, so it is important to maintain respiratory isolation as well as standard precautions. Because the patient may be confused and weak, bedrails should be elevated at both the food and head of the bed. Low light levels in the room decrease pain caused by photophobia. Nutrition is an important aspect of care in a patient with meningitis. Cognitive Level: Application Text Reference: p. 1496 Nursing Process: Assessment NCLEX: Safe and Effective Care Environment

24. While admitting a patient with a basal skull fracture, the nurse notes clear drainage from the patient's nose. Which of these admission orders should the nurse question? a. Insert nasogastric tube. b. Turn patient every 2 hours. c. Keep head of bed elevated. d. Cold packs for facial bruising.

Correct Answer: A Rationale: Rhinorrhea may indicate a dural tear with cerebrospinal fluid (CSF) leakage, and insertion of a nasogastric tube will increase the risk for infections such as meningitis. Turning the patient, elevation of the head, and cold pack applications are appropriate orders. Cognitive Level: Application Text Reference: p. 1486 Nursing Process: Implementation NCLEX: Physiological Integrity

20. An unconscious patient has a nursing diagnosis of ineffective tissue perfusion (cerebral) related to cerebral tissue swelling. An appropriate nursing intervention for this problem is to a. maintain the patient in a head-up position. b. position the patient with the knees and hips flexed. c. cluster nursing interventions to provide uninterrupted periods of rest. d. encourage coughing and deep-breathing to improve oxygenation.

Correct Answer: A Rationale: The patient with increased intracranial pressure (ICP) should be maintained in the head-up position to help reduce ICP. Flexion of the hips and knees increases abdominal pressure, which increases ICP. Because the stimulation associated with nursing interventions increases ICP, clustering interventions will progressively elevate ICP. Coughing increases intrathoracic pressure and ICP. Cognitive Level: Application Text Reference: p. 1480 Nursing Process: Implementation NCLEX: Physiological Integrity

23. A victim of an automobile accident was found unconscious at the scene of the accident but briefly regained consciousness during transport to the hospital. On admission, the Glasgow Coma Scale score is 8, and an acute epidural hematoma is suspected. The nurse will anticipate the need to a. prepare the patient for immediate craniotomy. b. administer IV furosemide (Lasix). c. type and crossmatch for blood transfusion. d. initiate high-dose barbiturate therapy.

Correct Answer: A Rationale: The principal treatment for epidural hematoma is rapid surgery to remove the hematoma and prevent herniation. If ICP is elevated after surgery, furosemide or high-dose barbiturate therapy may be needed, but these will not be of benefit unless the hematoma is removed. Minimal blood loss occurs with head injuries, and transfusion is usually not necessary. Cognitive Level: Application Text Reference: pp. 1483, 1485 Nursing Process: Planning NCLEX: Physiological Integrity

12. A patient is brought to the emergency department (ED) by ambulance after being found unconscious on the bathroom floor by the spouse. In admitting the patient, the nurse will first assess a. medication history. b. oxygen saturation. c. Glasgow Coma Scale (GCS). d. pupil reaction to light.

Correct Answer: B Rationale: Airway patency and breathing are the most vital functions and should be assessed first. The neurologic assessments should be accomplished next and the health and medication history last. Cognitive Level: Application Text Reference: p. 1476 Nursing Process: Assessment NCLEX: Physiological Integrity

14. The health care provider prescribes IV mannitol (Osmitrol) for an unconscious patient. The nurse will determine that the medication is effective if a. seizure behavior is reduced. b. intracranial pressure (ICP) is lower. c. abnormal electroencephalographic (EEG) activity decreases. d. Glasgow Coma score (GCS) is lower.

Correct Answer: B Rationale: Mannitol is an osmotic diuretic and will reduce cerebral edema and ICP. It will not directly affect seizure activity or abnormal EEG activity. A decreased GCS would indicate worsening of the patient's neurologic status. Cognitive Level: Application Text Reference: p. 1474 Nursing Process: Evaluation NCLEX: Physiological Integrity

30. Following a craniotomy with a craniectomy and left anterior fossae incision, the patient has a nursing diagnosis of impaired physical mobility related to decreased level of consciousness (LOC) and weakness. An appropriate nursing intervention is to a. position the bed flat and log roll the patient. b. perform range-of-motion (ROM) exercises every 4 hours. c. turn and reposition the patient side to side every 2 hours. d. cluster nursing activities to allow longer rest periods.

Correct Answer: B Rationale: ROM exercises will help to prevent the complications of immobility. Patients with anterior craniotomies are positioned with the head elevated. The patient with a craniectomy should not be turned to the operative side. When the patient is weak, clustering nursing activities may lead to more fatigue and weakness. Cognitive Level: Application Text Reference: pp. 1480, 1493 Nursing Process: Implementation NCLEX: Physiological Integrity

29. A patient with a brain tumor is receiving radiation after having had a craniotomy. The nurse will explain that the purpose of the ordered methylprednisolone (Solu-Medrol) is to a. eliminate the remaining tumor cells. b. prevent an increase in intracranial pressure (ICP). c. promote wound healing after the craniotomy. d. decrease the risk of metastasis of the cancer.

Correct Answer: B Rationale: Radiation can lead to cerebral edema and rapid ICP increases and corticosteroids are administered to prevent this. Corticosteroids do not damage tumor cells, promote wound healing, or decrease risk for metastasis. Cognitive Level: Application Text Reference: p. 1489 Nursing Process: Implementation NCLEX: Physiological Integrity

34. The community health nurse is developing a program to decrease the incidence of meningitis in adolescents and young adults. Which nursing action is most important? a. Emphasize the importance of hand washing to prevent spread of infection. b. Immunize adolescents and college freshman against Neisseria meningitides. c. Vaccinate 11 and 12 year-old children against Haemophilus influenzae. d. Encourage adolescents and young adults to avoid crowded areas in the winter.

Correct Answer: B Rationale: The Neisseria meningitides vaccination is recommended for children ages 11 and 12, unvaccinated teens entering high school, and college freshmen. Hand washing may help to decrease the spread of bacteria, but it is not as effective as immunization. Vaccination with Haemophilus influenzae is for infants and toddlers. Because adolescents and young adults are in school or the workplace, avoiding crowds is not realistic. Cognitive Level: Application Text Reference: p. 1495 Nursing Process: Implementation NCLEX: Health Promotion and Maintenance

16. When caring for a patient who has had a head injury, which assessment information is of most concern to the nurse? a. The blood pressure increases from 120/54 to 136/62. b. The patient is more difficult to arouse. c. The patient complains of a headache at pain level 5 of a 10-point scale. d. The patient's apical pulse is slightly irregular.

Correct Answer: B Rationale: The change in level of consciousness (LOC) is an indicator of increased ICP and suggests that action by the nurse is needed to prevent complications. The change in BP should be monitored but is not an indicator of a need for immediate nursing action. Headache is not unusual in a patient after a head injury. A slightly irregular apical pulse is not unusual. Cognitive Level: Application Text Reference: p. 1470 Nursing Process: Assessment NCLEX: Physiological Integrity

17. A patient with a head injury opens the eyes to verbal stimulation, curses when stimulated, and does not respond to a verbal command to move but attempts to remove a painful stimulus. The nurse records the patient's Glasgow Coma Scale score as a. 9. b. 11. c. 13. d. 15.

Correct Answer: B Rationale: The patient has a score of 3 for eye opening, 3 for best verbal response, and 5 for best motor response. Cognitive Level: Application Text Reference: p. 1477 Nursing Process: Assessment

4. A patient with possible cerebral edema has a serum sodium level of 115 mEq/L (115 mmol/L), a decreasing level of consciousness (LOC) and complains of a headache. All of the following orders have been received. Which one should the nurse accomplish first? a. Administer acetaminophen (Tylenol) 650 mg orally. b. Administer 5% hypertonic saline intravenously. c. Draw blood for arterial blood gases (ABGs). d. Send patient to radiology for computed tomography (CT) of the head.

Correct Answer: B Rationale: The patient's low sodium indicates that hyponatremia may be causing the cerebral edema, and the nurse's first action should be to correct the low sodium level. Acetaminophen (Tylenol) will have minimal effect on the headache because it is caused by cerebral edema and increased ICP. Drawing ABGs and obtaining a CT scan may add some useful information, but the low sodium level may lead to seizures unless it is addressed quickly. Cognitive Level: Application Text Reference: p. 1470 Nursing Process: Implementation NCLEX: Physiological Integrity

28. Four days after a patient has undergone a craniotomy to remove an astrocytoma of the temporal lobe, the dressing is removed and the nurse finds the patient crying. The patient tells the nurse, "I look awful and feel even worse." The most appropriate nursing diagnosis for the patient is a. grieving related to the patient's ongoing fear of dying. b. disturbed body image related to postoperative change in appearance. c. ineffective denial related to unrealistic expectations about surgery. d. hopelessness related to emotional lability secondary to cerebral edema.

Correct Answer: B Rationale: The patient's statement about looking and feeling awful supports the diagnosis of disturbed body image, which is common after surgery because of shaving of the scalp, incisions, and dressings, etc. There is no indication that the patient's immediate concern is with dying. The patient does not have indications of denial. The assessment data do not indicate that the patient feels hopeless or that cerebral edema is contributing to the patient's emotional status. Cognitive Level: Application Text Reference: p. 1493 Nursing Process: Diagnosis NCLEX: Psychosocial Integrity

27. A patient with increasing headaches who is having diagnostic testing for a brain tumor asks the nurse what type of treatment will be used if a tumor is discovered. Which response by the nurse is most appropriate? a. "If the tumor is benign, treatment may not be necessary." b. "Therapy to remove or reduce the tumor size will be recommended." c. "Surgery will initially be used to reduce or remove the tumor." d. "Chemotherapy is used to shrink the tumor, followed by craniotomy."

Correct Answer: B Rationale: Treatment is designed to reduce tumor size or remove the tumor. Benign brain tumors place pressure on intracranial structures and need to be treated. Surgery is the preferred initial therapy but may not be possible for tumors located deep in the brain. The usefulness of chemotherapy is limited in brain tumors; chemotherapy is usually not the initial treatment. Cognitive Level: Application Text Reference: p. 1489 Nursing Process: Implementation NCLEX: Physiological Integrity

8. When caring for a patient with a right-sided intracerebral hemorrhage, the nurse suspects possible supratentorial herniation and compression of the brainstem when the a. corneal reflexes are absent. b. patient develops nystagmus. c. right pupil does not react to light. d. left pupil is 10 mm in size.

Correct Answer: C Rationale: A dilated pupil on the ipsilateral side in a patient with an acute brain injury indicates herniation. Absent corneal reflexes and nystagmus are not symptoms of herniation. A nonreactive left pupil would be consistent with left-sided damage. Cognitive Level: Application Text Reference: p. 1471 Nursing Process: Assessment NCLEX: Physiological Integrity

22. A patient is brought to the emergency department (ED) after being hit in the head with a baseball during a company picnic. On admission, the patient has a headache and cannot remember being hit but has no other signs of neurologic deficit. The nurse will plan to a. send the patient for diagnostic testing with MRI. b. admit the patient for observation for 24 hours. c. discharge the patient with monitoring instructions. d. observe the patient in the ED for several hours.

Correct Answer: C Rationale: A patient with a minor head trauma is usually discharged with instructions about neurologic monitoring and the need to return if neurologic status deteriorates. MRI, hospital admission, and continued observation in the ED are not indicated in a patient who only briefly lost consciousness and has no neurologic deficits. Cognitive Level: Application Text Reference: p. 1482 Nursing Process: Implementation NCLEX: Physiological Integrity

3. Which of these patients is most appropriate for the intensive care unit (ICU) charge nurse to assign to an RN who has floated from the medical unit? a. A 23-year-old patient who had a skull fracture and craniotomy the previous day b. A 30-year-old patient who has an ICP monitor in place after a head injury a week ago c. A 44-year-old patient receiving IV antibiotics for meningococcal meningitis d. A 61-year-old patient who has increased ICP and is receiving hyperventilation therapy

Correct Answer: C Rationale: An RN who works on a medical unit will be familiar with administration of IV antibiotics and with meningitis. The post-craniotomy patient, patient with an ICP monitor, and the patient on a ventilator should be assigned to an RN familiar with the care of critically ill patients. Cognitive Level: Application Text Reference: pp. 1495-1497 Nursing Process: Implementation NCLEX: Safe and Effective Care Environment

25. In planning discharge for the patient following brain trauma, the nurse includes teaching and support for the family, primarily because a. the residual deficits of the brain damage are unlikely to improve in the months after discharge. b. families become dysfunctional and unable to cope with the role reversals required during convalescence. c. patients with severe head injuries often have changes in personality with loss of concentration and memory processing. d. most patients experience seizure disorders in the weeks or even years following head injury.

Correct Answer: C Rationale: Changes in personality, concentration, and memory are common after severe head injury and require anticipatory guidance for the patient and family. Recovery continues for up to 6 months after the injury. Most families are able to cope with the changes in role during the convalescence. Seizure disorders are more common soon after brain injury, and most patients do not develop seizures. Cognitive Level: Application Text Reference: p. 1487 Nursing Process: Implementation NCLEX: Psychosocial Integrity

21. The nurse notes that a patient with a head injury has a clear nasal drainage. The most appropriate nursing action for this finding is to a. obtain a specimen of the fluid and send for culture and sensitivity. b. take the patient's temperature to determine whether a fever is present. c. check the nasal drainage for glucose with a Dextrostik or Testape. d. have the patient to blow the nose and then check the nares for redness.

Correct Answer: C Rationale: If the drainage is cerebrospinal fluid (CSF) leakage from a dural tear, glucose will be present. Fluid leaking from the nose will have normal nasal flora, so culture and sensitivity will not be useful. A dural tear does increase the risk for infections such as meningitis, but the nurse should first determine whether the clear drainage is CSF. Blowing the nose is avoided to prevent CSF leakage. Cognitive Level: Application Text Reference: p. 1481 Nursing Process: Implementation NCLEX: Physiological Integrity

6. When assessing a patient with a head injury, the nurse recognizes that the earliest indication of increased intracranial pressure (ICP) is a. vomiting. b. headache. c. change in level of consciousness (LOC). d. sluggish pupil response to light.

Correct Answer: C Rationale: LOC is the most sensitive indicator of the patient's neurologic status and possible changes in ICP. Vomiting and sluggish pupil response to light are later signs of increased ICP. A headache can be caused by compression of intracranial structures as the brain swells, but it is not unexpected after a head injury. Cognitive Level: Comprehension Text Reference: p. 1470 Nursing Process: Assessment NCLEX: Physiological Integrity

31. A patient who has bacterial meningitis and is disoriented and anxious has a nursing diagnosis of disturbed sensory perception related to decreased level of consciousness. An appropriate nursing intervention is to a. apply soft restraints to protect the patient from injury. b. minimize contact with the patient to decrease sensory input. c. encourage family members to remain at the bedside. d. keep the room well-lighted to improve patient orientation.

Correct Answer: C Rationale: Patients with meningitis and disorientation will be calmed by the presence of someone familiar at the bedside. Restraints should be avoided because they increase agitation and anxiety. The patient requires frequent assessment for complications; the use of touch and a soothing voice will decrease anxiety for most patients. The patient will have photophobia, so the light should be dim. Cognitive Level: Application Text Reference: p. 1495 Nursing Process: Implementation NCLEX: Physiological Integrity

32. When assessing a patient with bacterial meningitis, the nurse obtains all of the following information. Which should be reported immediately to the health care provider? a. The patient complains of having a stiff neck. b. The patient has a positive Kernig's sign. c. The patient's blood pressure is 86/42 mm Hg. d. The patient's temperature is 102° F.

Correct Answer: C Rationale: Shock is a serious complication of meningitis, and the patient's low blood pressure indicates the need for interventions such as fluids or vasopressors. Nuchal rigidity and a positive Kernig's sign are expected with bacterial meningitis. The nurse should intervene to lower the temperature, but this is not as life threatening as the hypotension. Cognitive Level: Application Text Reference: p. 1494 Nursing Process: Assessment NCLEX: Physiological Integrity

7. A patient admitted with a head injury has admission vital signs of temperature 98.6° F (37° C), blood pressure 128/68, pulse 110, and respirations 26. Which of these vital signs, if taken 1 hour after admission, will be of most concern to the nurse? a. Blood pressure 130/72, pulse 90, respirations 32 b. Blood pressure 148/78, pulse 112, respirations 28 c. Blood pressure 156/60, pulse 60, respirations 14 d. Blood pressure 110/70, pulse 120, respirations 30

Correct Answer: C Rationale: Systolic hypertension with widening pulse pressure, bradycardia, and respiratory changes represent Cushing's triad and indicate that the ICP has increased and brain herniation may be imminent unless immediate action is taken to reduce ICP. The other vital signs may indicate the need for changes in treatment, but they are not indicative of an immediately life-threatening process. Cognitive Level: Application Text Reference: p. 1469 Nursing Process: Assessment NCLEX: Physiological Integrity

26. When admitting a patient who has a tumor of the right frontal lobe, the nurse would expect to find a. expressive aphasia. b. right-sided weakness. c. judgment changes. d. difficulty swallowing.

Correct Answer: C Rationale: The frontal lobes control intellectual activities such as judgment. Speech is controlled in the parietal lobe. Weakness and hemiplegia occur on the contralateral side from the tumor. Swallowing is controlled by the brainstem. Cognitive Level: Application Text Reference: pp. 1488-1489 Nursing Process: Assessment NCLEX: Physiological Integrity

19. A patient is admitted unconscious to the emergency department (ED) after falling and hitting the head on a rock while hiking. The patient's spouse and children stay at the patient's side and constantly ask about the treatment being given. The nurse's best approach to the patient's family is to a. call the family's pastor or spiritual advisor to support them while initial care is given. b. refer the family members to the hospital counseling service to deal with their anxiety. c. allow the family to stay with the patient and explain all procedures thoroughly to them. d. ask the family to stay in the waiting room while the initial assessment and care are done.

Correct Answer: C Rationale: The need for information about the diagnosis and care is very high in family members of acutely ill patients, and the nurse should allow the family to observe care and explain the procedures. A pastor or counseling service can offer some support, but research supports information as being more effective. Asking the family to stay in the waiting room will increase their anxiety. Cognitive Level: Application Text Reference: p. 1486 Nursing Process: Implementation NCLEX: Psychosocial Integrity

15. A patient with a severe head injury has been maintained on IV fluids of 5% dextrose in water (D5W) at 50 ml/hr for 3 days. The nurse will anticipate the need to a. continue the D5W to provide the needed glucose for brain function. b. decrease the rate of IV infusion to avoid increasing cerebral edema. c. insert an enteral feeding tube to provide nutritional replacement. d. administer IV 5% albumin to increase serum protein levels.

Correct Answer: C Rationale: The patient is in a hypermetabolic and hypercatabolic state, and enteral feedings will provide nutrients for brain function and also for healing and immune function. 5% dextrose does not provide adequate nutrition to meet patient needs and can lead to lower serum osmolarity and cerebral edema. A total fluid intake of 1200 ml for 24 hours will not cause cerebral edema. Albumin administration will temporarily increase serum protein, but the patient also requires lipids, carbohydrate, and other nutrients that will be supplied through enteral feeding. Cognitive Level: Application Text Reference: p. 1475 Nursing Process: Planning NCLEX: Physiological Integrity

1. A patient has a systemic blood pressure (BP) of 120/60 mm Hg and an intracranial pressure of 24 mm Hg. The nurse determines that the cerebral perfusion pressure (CPP) of this patient indicates a. high blood flow to the brain. b. normal intracranial pressure (ICP). c. impaired brain blood flow. d. adequate cerebral perfusion.

Correct Answer: C Rationale: The patient's CPP is 56, below the normal of 70 to 100 mm Hg and approaching the level of ischemia and neuronal death. The patient has low cerebral blood flow/perfusion. Normal ICP is 0 to 15 mm Hg.

2. A patient with a head injury has an arterial blood pressure is 92/50 mm Hg and an intracranial pressure of 18 mm Hg. Which action by the nurse is appropriate? a. Document and continue to monitor the parameters. b. Elevate the head of the patient's bed. c. Notify the health care provider about the assessments. d. Check the patient's pupillary response to light.

Correct Answer: C Rationale: The patient's cerebral perfusion pressure is only 46 mm Hg, which will rapidly lead to cerebral ischemia and neuronal death unless rapid action is taken to reduce ICP and increase arterial BP. Documentation and monitoring are inadequate responses to the patient's problem. Elevating the head of the bed will lower the ICP but may also lower cerebral blood flow and further decrease CPP. Changes in pupil response to light are signs of increased ICP, so the nurse will only take more time doing this without adding any useful information. Cognitive Level: Analysis Text Reference: pp. 1468-1469 Nursing Process: Implementation NCLEX: Physiological Integrity

13. Mechanical ventilation with a rate and volume to maintain a mild hyperventilation is used for a patient with a head injury. To evaluate the effectiveness of the therapy, the nurse should a. monitor oxygen saturation. b. check arterial blood gases (ABGs). c. monitor intracranial pressure (ICP). d. assess patient breath sounds.

Correct Answer: C Rationale: The purpose of hyperventilation for a patient with a head injury is reduction of ICP, and ICP should be monitored to evaluate whether the therapy is effective. Although oxygen saturation and ABGs are monitored in patient's receiving hyperventilation, they do not provide data about whether the therapy is successful in reducing ICP. Breath sounds are assessed, but they are not helpful in determining whether the hyperventilation is effective. Cognitive Level: Application Text Reference: p. 1475 Nursing Process: Evaluation NCLEX: Physiological Integrity

33. A patient admitted with bacterial meningitis and a temperature of 102° F has orders for all of these collaborative interventions. Which one should the nurse accomplish first? a. IV ceftizoxime (Cefizox) 1 g now and every 6 hours b. IV dexamethasone (Decadron) 4 mg now c. Hypothermia blanket to keep temperature less than 101.6° F d. Nasopharyngeal swab for culture and sensitivity

Correct Answer: D Rationale: Antibiotic therapy should be instituted rapidly in bacterial meningitis, but cultures must be done before antibiotics are started. As soon as the cultures are done, the antibiotic should be started. Administration of dexamethasone and initiation of hypothermia therapy should be done as quickly as possible once cultures and antibiotics are initiated. Cognitive Level: Application Text Reference: p. 1495 Nursing Process: Implementation NCLEX: Physiological Integrity

5. Family members are optimistic about a comatose patient's recovery because the patient's eyes open and the patient appears to be awake at times. Which statement by the nurse to the family is appropriate? a. "The behavior is only a reflex and does not indicate improvement in the comatose condition." b. "Sleep-wake cycles are indicators of recovery and a sign that the brain function is improving." c. "When patients begin to recover from a coma, the first behaviors seen are those of wakefulness and opening the eyes." d. "The part of the brain responsible for arousal is not injured, but the wakefulness does not indicate improvement in higher brain centers."

Correct Answer: D Rationale: Arousal is controlled by the reticular activating system in the brainstem and will allow the patient to maintain wakefulness even though the damage to the cerebral cortex is severe. The patient's behavior is not a reflex action. The behaviors of eye opening and wakefulness are not indicators of improvement in the comatose condition. Cognitive Level: Application Text Reference: p. 1470 Nursing Process: Implementation NCLEX: Physiological Integrity

A patient is suspected of having a brain tumor. The s/s include: memory deficits, visual disturbances, weakness of right upper and lower extremities and personality changes. The nurse recognizes that the tumor is most likely located in the a. Frontal lobe b. Parietal lobe c. Occipital lobe d. Temporal lobe

Correct answer: a Rationale: A unilateral frontal lobe tumor may result in the following signs and symptoms: unilateral hemiplegia, seizures, memory deficit, personality and judgment changes, and visual disturbances. A bilateral frontal lobe tumor may cause symptoms associated with a unilateral frontal lobe tumor and an ataxic gait.

A patient is suspected of having a brain tumor. The s/s include: memory deficits, visual disturbances, weakness of right upper and lower extremities and personality changes. The nurse recognizes that the tumor is most likely located in the a. Frontal lobe b. Parietal lobe c. Occipital lobe d. Temporal lobe

Correct answer: a Rationale: A unilateral frontal lobe tumor may result in the following signs and symptoms: unilateral hemiplegia, seizures, memory deficit, personality and judgment changes, and visual disturbances. A bilateral frontal lobe tumor may cause symptoms associated with a unilateral frontal lobe tumor and an ataxic gait.

During admission of a patient with a severe head injury to the emergency department the nurse places the highest priority on assessment for a. Patency of airway b. Presence of neck injury c. Neurologic status with Glasgow coma scale d. Cerebrospinal fluid leakage from the ears or nose

Correct answer: a Rationale: The nurse's initial priority in the emergency management of a patient with a severe head injury is to ensure that the patient has a patent airway

During admission of a patient with a severe head injury to the emergency department the nurse places the highest priority on assessment for a. Patency of airway b. Presence of neck injury c. Neurologic status with Glasgow coma scale d. Cerebrospinal fluid leakage from the ears or nose

Correct answer: a Rationale: The nurse's initial priority in the emergency management of a patient with a severe head injury is to ensure that the patient has a patent airway.

A nursing measure that is indicated to reduce the potential for seizures and increased intracranial pressure in the patient with bacterial meningitis is a. Administering codeine for relief of head and neck pain b. Controlling fever with prescribed drugs and cooling techniques c. Keeping the room dark and quiet to minimize environmental stimulation d. Maintain the patient on strict bed rest with the head of the bed slightly elevated

Correct answer: b Rationale: Fever must be vigorously managed because it increases cerebral edema and the frequency of seizures. Neurologic damage may result from an extremely high temperature over a prolonged period. Acetaminophen or aspirin may be used to reduce fever; other measures, such as a cooling blanket or tepid sponge baths with water, may be effective in lowering the temperature.

A nursing measure that is indicated to reduce the potential for seizures and increased intracranial pressure in the patient with bacterial meningitis is a. Administering codeine for relief of head and neck pain b. Controlling fever with prescribed drugs and cooling techniques c. Keeping the room dark and quiet to minimize environmental stimulation d. Maintain the patient on strict bed rest with the head of the bed slightly elevated

Correct answer: b Rationale: Fever must be vigorously managed because it increases cerebral edema and the frequency of seizures. Neurologic damage may result from an extremely high temperature over a prolonged period. Acetaminophen or aspirin may be used to reduce fever; other measures, such as a cooling blanket or tepid sponge baths with water, may be effective in lowering the temperature.

A nurse plan care for the patient with increased intracranial pressure with the knowledge that the best way to position the patient is to a. Keep the head of the bed flat b. Elevate the HOB by 30 degrees c. Maintain pt on the left side with the head supported on a pillow d. Use a continuous rotation bed to continuously change patient position

Correct answer: b Rationale: The nurse should maintain the patient with abnormal ICP in the head-up position. Elevation of the head of the bed to 30 degrees enhances respiratory exchange and aids in decreasing cerebral edema. The nurse should position the patient to prevent extreme neck flexion, which can cause venous obstruction and contribute to elevation in ICP. Elevation of the head of the bed also reduces sagittal sinus pressure, promotes drainage from the head through the valveless venous system and jugular veins, and decreases the vascular congestion that can produce cerebral edema. However, raising the head of the bed above 30 degrees may decrease the cerebral perfusion pressure (CPP) by lowering systemic blood pressure. The effects of elevation of the head of the bed on the ICP and CPP must be evaluated carefully.

A nurse plans care for the patient with increased intracranial pressure with the knowledge that the best way to position the patient is to a. Keep the head of the bed flat b. Elevate the HOB by 30 degrees c. Maintain pt on the left side with the head supported on a pillow d. Use a continuous rotation bed to continuously change patient position

Correct answer: b Rationale: The nurse should maintain the patient with abnormal ICP in the head-up position. Elevation of the head of the bed to 30 degrees enhances respiratory exchange and aids in decreasing cerebral edema. The nurse should position the patient to prevent extreme neck flexion, which can cause venous obstruction and contribute to elevation in ICP. Elevation of the head of the bed also reduces sagittal sinus pressure, promotes drainage from the head through the valveless venous system and jugular veins, and decreases the vascular congestion that can produce cerebral edema. However, raising the head of the bed above 30 degrees may decrease the cerebral perfusion pressure (CPP) by lowering systemic blood pressure. The effects of elevation of the head of the bed on the ICP and CPP must be evaluated carefully.

Vasogenic cerebral edema increases intracranial pressure by A. Shifting fluid in the gray matter B. Altering the endothelial lining of the cerebral capillaries C. Leaking molecules from the intercellular fluid to the capillaries D. Altering the osmotic gradient flow into the intravascular component

Correct answer: b Rationale: Vasogenic cerebral edema occurs mainly in the white matter. It is caused by changes in the endothelial lining of cerebral capillaries.

Vasogenic cerebral edema increases intracranial pressure by A. Shifting fluid in the gray matter B. Altering the endothelial lining of the cerebral capillaries C. Leaking molecules from the intercellular fluid to the capillaries D. Altering the osmotic gradient flow into the intravascular component

Correct answer: b Rationale: Vasogenic cerebral edema occurs mainly in the white matter. It is caused by changes in the endothelial lining of cerebral capillaries.

The nurse is alerted to a possible acute subdural hematoma in the patient who a. Has linear skull fracture crossing a major artery b. Has focal symptoms of brain damage with no recollection of a head injury c. Develops decreased level of consciousness and a headache within 48 hrs of a head injury d. Has an immediate loss of consciousness with brief lucid interval followed by dereasing level of consciousness

Correct answer: c Rationale: An acute subdural hematoma manifests within 24 to 48 hours of the injury. The signs and symptoms are similar to those associated with brain tissue compression in elevated ICP and include decreasing level of consciousness and headache

The nurse is altered to a possible acute subdural hematoma in the patient who a. Has linear skull fracture crossing a major artery b. Has focal symptoms of brain damage with no recollection of a head injury c. Develops decreased level of consciousness and a headache within 48 hrs of a head injury d. Has an immediate loss of consciousness with brief lucid interval followed by dereasing level of consciousness

Correct answer: c Rationale: An acute subdural hematoma manifests within 24 to 48 hours of the injury. The signs and symptoms are similar to those associated with brain tissue compression in elevated ICP and include decreasing level of consciousness and headache

Nurse on clinical unit is assigned to four patients. Which patient should she assess first? a. Patient with a skull fracture whose nose is bleeding b. Older patient with a stroke who is confused and whose daughter is present c. Patient with meningitis who is suddenly agitated and reporting a headache of 10 on a 0-10 scale d. Patient who had a craniotomy for a brain tumor who now 3 days postoperative had had continued vomiting

Correct answer: c Rationale: The patient with meningitis should be seen first; patients with meningitis must be observed closely for manifestations of elevated ICP, which is thought to result from swelling around the dura and increased cerebrospinal fluid (CSF) volume. Sudden change in the level of consciousness or change in behavior along with a sudden severe headache may indicate an acute elevation of ICP. The patient who has undergone cranial surgery should be seen second; although nausea and vomiting are common after cranial surgery, it can result in elevations of ICP. Nausea and vomiting should be treated with antiemetics. The patient with a skull fracture needs to be evaluated for CSF leakage occurring with the nose bleed and should be seen third. Confusion after a stroke may be expected; the patient should have a family member present.

The nurse on clinical unit is assigned to four patients. Which patient should she assess first? a. Patient with a skull fracture whose nose is bleeding b. Older patient with a stroke who is confused and whose daughter is present c. Patient with meningitis who is suddenly agitated and reporting a headache of 10 on a 0-10 scale d. Patient who had a craniotomy for a brain tumor who now 3 days postoperative had had continued vomiting

Correct answer: c Rationale: The patient with meningitis should be seen first; patients with meningitis must be observed closely for manifestations of elevated ICP, which is thought to result from swelling around the dura and increased cerebrospinal fluid (CSF) volume. Sudden change in the level of consciousness or change in behavior along with a sudden severe headache may indicate an acute elevation of ICP. The patient who has undergone cranial surgery should be seen second; although nausea and vomiting are common after cranial surgery, it can result in elevations of ICP. Nausea and vomiting should be treated with antiemetics. The patient with a skull fracture needs to be evaluated for CSF leakage occurring with the nose bleed and should be seen third. Confusion after a stroke may be expected; the patient should have a family member present.

A patient with intracranial pressure monitoring has a pressure of 12mm Hg. The nurse understands that this pressure reflects a. A severe decrease in cerebral perfusion pressure b. An alteration in the production of cerebrospinal fluid c. The loss of autoregulatory control of intracranial pressure d. A normal balance between brain tissue, blood and cerebrospinal fluid

Correct answer: d Rationale: Normal intracranial pressure (ICP) is 5 to 15 mm Hg. A sustained pressure above the upper limit is considered abnormal.

A patient with intracranial pressure monitoring has a pressure of 12mm Hg. The nurse understands that this pressure reflects a. A severe decrease in cerebral perfusion pressure b. An alteration in the production of cerebrospinal fluid c. The loss of autoregulatory control of intracranial pressure d. A normal balance between brain tissue, blood and cerebrospinal fluid

Correct answer: d Rationale: Normal intracranial pressure (ICP) is 5 to 15 mm Hg. A sustained pressure above the upper limit is considered abnormal.

A male patient suffered a diffuse axonal injury from a traumatic brain injury (TBI). He has been maintained on IV fluids for 2 days. The nurse seeks enteral feeding for this patient based on what rationale? A. Free water should be avoided. B. Sodium restrictions can be managed. C. Dehydration can be better avoided with feedings. D. Malnutrition promotes continued cerebral edema.

D A patient with diffuse axonal injury is unconscious and, with increased ICP, is in a hypermetabolic, hypercatabolic state that increases the need for fuel for healing. Malnutrition promotes continued cerebral edema, and early feeding may improve outcomes when begun within 3 days after injury. Fluid and electrolytes will be monitored to maintain balance with the enteral feedings.

A male patient suffered a diffuse axonal injury from a traumatic brain injury (TBI). He has been maintained on IV fluids for 2 days. The nurse seeks enteral feeding for this patient based on what rationale? A. Free water should be avoided. B. Sodium restrictions can be managed. C. Dehydration can be better avoided with feedings. D. Malnutrition promotes continued cerebral edema.

D A patient with diffuse axonal injury is unconscious and, with increased ICP, is in a hypermetabolic, hypercatabolic state that increases the need for fuel for healing. Malnutrition promotes continued cerebral edema, and early feeding may improve outcomes when begun within 3 days after injury. Fluid and electrolytes will be monitored to maintain balance with the enteral feedings.

A 32-year-old female patient is diagnosed with diabetes insipidus after transsphenoidal resection of a pituitary adenoma. What should the nurse consider as a sign of improvement? A. Serum sodium of 120 mEq/L B. Urine specific gravity of 1.001 C. Fasting blood glucose of 80 mg/dL D. Serum osmolality of 290 mOsm/kg

D Laboratory findings in diabetes insipidus include an elevation in serum osmolality and serum sodium and a decrease in urine specific gravity. Normal serum osmolality is 275 to 295 mOsm/kg, normal serum sodium is 135 to 145 mEq/L, and normal specific gravity is 1.003 to 1.030. Elevated blood glucose levels occur with diabetes mellitus.

The nurse assesses a patient for signs of meningeal irritation and observes for nuchal rigidity. What indicates the presence of this sign of meningeal irritation? A. Tonic spasms of the legs B. Curling in a fetal position C. Arching of the neck and back D. Resistance to flexion of the neck

D Nuchal rigidity is a clinical manifestation of meningitis. During assessment, the patient will resist passive flexion of the neck by the health care provider. Tonic spasms of the legs, curling in a fetal position, and arching of the neck and back are not related to meningeal irritation.

The nurse assesses a patient for signs of meningeal irritation and observes for nuchal rigidity. What indicates the presence of this sign of meningeal irritation? A. Tonic spasms of the legs B. Curling in a fetal position C. Arching of the neck and back D. Resistance to flexion of the neck

D Nuchal rigidity is a clinical manifestation of meningitis. During assessment, the patient will resist passive flexion of the neck by the health care provider. Tonic spasms of the legs, curling in a fetal position, and arching of the neck and back are not related to meningeal irritation.

Metabolic and nutritional needs of the patient with increased ICP are best met with a. enteral feedings that are low in sodium b. the simple glucose available in D5W IV solutions c. a fluid restriction that promotes a moderate dehydration d. balanced, essential nutrition in a form that the patient can tolerate

D. A patient with increased ICP is in a hypermetabolic and hypercatabolic state and needs adequate glucose to maintain fuel for the brain and other nutrients to meet metabolic needs. Malnutrition promotes cerebral edema, and if a patient cannot take oral nutrition, other means of providing nutrition should be used, such as tube feedings or parenteral nutrition. Glucose alone is not adequate to meet nutritional requirements, and 5% dextrose solutions may increase cerebral edema by lowering serum osmolarity. Patients should remain in a normovolemic fluid state with close monitoring of clinical factors such as urine output, fluid intake, serum and urine osmolality, serum electrolytes, and insensible losses.

The nurse suspects the presence of an arterial epidural hematoma in the patient who experiences a. failure to regain consciousness following a head injury b. a rapid deterioration of neurologic function within 24 to 48 hours following a head injury c. nonspecific, nonlocalizing progression of alteration in LOC occurring over weeks or months d. unconsciousness at the time of a head injury with a brief period of consciousness followed by a decrease in LOC

D. An arterial epidural hematoma is the most acute neurologic emergency, and the typical symptoms include unconsciousness at the scene, with a brief lucid interval followed by a decrease in LOC. An acute subdural hematoma manifests signs within 48 hours of an injury; a chronic subdural hematoma develops over weeks or months

Which medication taken by a patient with restless legs syndrome should the nurse discuss with the patient? a. Multivitamin (Stresstabs) b. Acetaminophen (Tylenol) c. Ibuprofen (Motrin, Advil) d. Diphenhydramine (Benadryl)

D. Antihistamines can aggravate restless legs syndrome. The other medications will not contribute to restless legs syndrome.

Which information about a 60-year-old patient with MS indicates that the nurse should consult with the health care provider before giving the prescribed dose of dalfampridine (Ampyra)? a. The patient has relapsing-remitting MS. b. The patient walks a mile a day for exercise. c. The patient complains of pain with neck flexion. d. The patient has an increased serum creatinine level.

D. Dalfampridine should not be given to patients with impaired renal function. The other information will not impact whether the dalfampridine should be administered.

A 76-year-old patient is being treated with carbidopa/levodopa (Sinemet) for Parkinson's disease. Which information is most important for the nurse to report to the health care provider? a. Shuffling gait b. Tremor at rest c. Cogwheel rigidity of limbs d. Uncontrolled head movement

D. Dyskinesia is an adverse effect of the Sinemet, indicating a need for a change in medication or decrease in dose. The other findings are typical with Parkinson's disease.

A 62-year-old patient who has Parkinson's disease is taking bromocriptine (Parlodel). Which information obtained by the nurse may indicate a need for a decrease in the dose? a. The patient has a chronic dry cough. b. The patient has four loose stools in a day. c. The patient develops a deep vein thrombosis. d. The patient's blood pressure is 92/52 mm Hg.

D. Hypotension is an adverse effect of bromocriptine, and the nurse should check with the health care provider before giving the medication. Diarrhea, cough, and deep vein thrombosis are not associated with bromocriptine use.

A patient is admitted to the hospital with possible bacterial meningitis. During the initial assessment, the nurse questions the patient about a recent history of a. mosquito or tick bites b. chickenpox or measles c. cold sores or fever blisters d. an upper respiratory infection

D. Meningitis is often a result of an upper respiratory infection or middle ear infection, where organisms gain entry to the CNS. Epidemic encephalitis is transmitted by ticks and mosquitoes, and nonepidemic encephalitis may occur as a complication of measles, chickenpox, or mumps. Encephalitis caused by the herpes simplex virus carries a high fatality rate

When assessing the body function of a patient with increased ICP, the nurse should initially assess a. corneal reflex testing b. extremity strength testing c. pupillary reaction to light d. circulatory and respiratory status

D. Of the body functions that should be assessed in an unconscious patient, cardiopulmonary status is the most vital function and gives priorities to the ABCs (airway, breathing, and circulation).

Which nursing diagnosis is of highest priority for a patient with Parkinson's disease who is unable to move the facial muscles? a. Activity intolerance b. Self-care deficit: toileting c. Ineffective self-health management d. Imbalanced nutrition: less than body requirements

D. The data about the patient indicate that poor nutrition will be a concern because of decreased swallowing. The other diagnoses may also be appropriate for a patient with Parkinson's disease, but the data do not indicate that they are current problems for this patient.

Successful achievement of patient outcomes for the patient with cranial surgery would be best indicated by the a. ability to return home in 6 days b. ability to meet all self-care needs c. acceptance of residual neurologic deficits d. absence of signs and symptoms of increased ICP

D. The primary goal after cranial surgery is prevention of increased ICP, and interventions to prevent ICP and infection postoperatively are nursing priorities. The residual deficits, rehabilitation potential, and ultimate function of the patient depend on the reason for surgery, the postoperative course, and the patient's general state of health

Skull radiographs and a computed tomography (CT) scan provide evidence of a depressed parietal fracture with a subdural hematoma in a patient admitted to the emergency department following an automobile accident. In planning care for the patient, the nurse anticipates that a. the patient will receive life-support measures until the condition stabilizes b. immediate burr holes will be made to rapidly decompress the intracranial activity c. the patient will be treated conservatively with close monitoring for changes in neurologic condition d. the patient will be taken to surgery for a craniotomy for evacuation of blood and decompression of the cranium

D. When there is a depressed fracture and fractures with loose fragments, a craniotomy is indicated to elevate the depressed bone and remove free fragments. A craniotomy is also indicated in cases of acute subdural and epidural hematomas to remove the blood and control the bleeding. Burr holes may be used in an extreme emergency for rapid decompression, but with a depressed fracture, surgery would be the treatment of choice

A patient is having a transsphenoidal hypophysectomy. The nurse should provide preoperative patient teaching about what potential deficit as a result of the surgery? A. Increased heart rate B. Loss of coordination C. Impaired swallowing D. Altered sense of smell

D. Altered sense of smell Using a transsphenoidal approach to remove the pituitary gland includes a risk of damage to the olfactory cranial nerve because the cell bodies of the olfactory nerve are located in the nasal epithelium. With damage to this nerve, the sense of smell would be altered. Increased heart rate, loss of coordination, and impaired swallowing will not be potential deficits from this surgery.

When assessing motor function of a patient admitted with a stroke, the nurse notes mild weakness of the arm demonstrated by downward drifting of the arm. How should the nurse most accurately document this finding? A. Athetosis B. Hypotonia C. Hemiparesis D. Pronator drift

D. Pronator drift Downward drifting of the arm or pronation of the palm is identified as a pronator drift. Athetosis is a slow, writhing, involuntary movement of the extremities. Hypotonia is flaccid muscle tone, and hemiparesis is weakness of one side of the body.

A 50-year-old man complains of recurring headaches. He describes these as sharp, stabbing, and located around his left eye. He also reports that his left eye seems to swell and get teary when these headaches occur. Based on this history, you suspect that he has a. cluster headaches. b. tension headaches. c. migraine headaches. d. medication overuse headaches.

a Rationale: Cluster headaches involve repeated headaches that can occur for weeks to months, followed by periods of remission. The pain of cluster headache is sharp and stabbing; the intense pain lasts a few minutes to 3 hours. Cluster headaches can occur every other day and as often as eight times a day. The clusters occur with regularity, usually occurring at the same time each day and during the same seasons of the year. Typically, a cluster lasts 2 weeks to 3 months, and the patient then goes into remission for months to years. The pain usually is located around the eye and radiates to the temple, forehead, cheek, nose, or gums. Other manifestations may include swelling around the eye, lacrimation (tearing), facial flushing or pallor, nasal congestion, and constriction of the pupil. During the headache, the patient is often agitated and restless, unable to sit still or relax.

When providing care for a patient with ALS, the nurse recognizes what as one of the most distressing problems experienced by the patient? a. Painful spasticity of the face and extremities b. Retention of cognitive function with total degeneration of motor function c. Uncontrollable writhing and twisting movements of the face, limbs, and body d. Knowledge that there is a 50% chance the disease has been passed to any offspring

b. In ALS there is gradual degeneration of motor neurons with extreme muscle wasting from lack of stimulation and use. However, cognitive function is not impaired and patients feel trapped in a dying body. Chorea manifested by writhing, involuntary movements is characteristic of HD. As an autosomal dominant genetic disease, HD also has a 50% chance of being passed to each offspring.

In providing care for patients with chronic, progressive neurologic disease, what is the major goal of treatment that the nurse works toward? a. Meet the patient's personal care needs. b. Return the patient to normal neurologic function. c. Maximize neurologic functioning for as long as possible. d. Prevent the development of additional chronic diseases.

c. Many chronic neurologic diseases involve progressive deterioration in physical or mental capabilities and have no cure, with devastating results for patients and families. Health care providers can only attempt to alleviate physical symptoms, prevent complications, and assist patients in maximizing function and self-care abilities for as long as possible.

A patient with myasthenia gravis is admitted to the hospital with respiratory insufficiency and severe weakness. When is a diagnosis of cholinergic crisis made? a. The patient's respiration is impaired because of muscle weakness. b. Administration of edrophonium (Tensilon) increases muscle weakness. c. Administration of edrophonium (Tensilon) results in improved muscle contractility. d. EMG reveals decreased response to repeated stimulation of muscles.

b. The reduction of the acetylcholine (ACh) effect in myasthenia gravis (MG) is treated with anticholinesterase drugs, which prolong the action of ACh at the neuromuscular synapse, but too much of these drugs will cause a cholinergic crisis with symptoms very similar to those of MG. To determine whether the patient's manifestations are due to a deficiency of ACh or to too much anticholinesterase drug, the anticholinesterase drug edrophonium chloride (Tensilon) is administered. If the patient is in cholinergic crisis, the patient's symptoms will worsen; if the patient is in a myasthenic crisis, the patient will improve.

A 65-year-old woman was just diagnosed with Parkinson's disease. The priority nursing intervention is a. searching the Internet for educational videos. b. evaluating the home for environmental safety. c. promoting physical exercise and a well-balanced diet. d. designing an exercise program to strengthen and stretch specific muscles.

c Rationale: Promotion of physical exercise and a well-balanced diet are major concerns of nursing care for patients with Parkinson's disease.

During care of a patient in myasthenic crisis, maintenance of what is the nurse's first priority for the patient? a. Mobility b. Nutrition c. Respiratory function d. Verbal communication

c. The patient in myasthenic crisis has severe weakness and fatigability of all skeletal muscles, affecting the patient's ability to breathe, swallow, talk, and move. However, the priority of nursing care is monitoring and maintaining adequate ventilation.

30. To reduce the risk for falls in the patient with Parkinson's disease, what should the nurse teach the patient to do? a. Use an elevated toilet seat. b. Use a walker or cane for support. c. Consciously lift the toes when stepping. d. Rock side to side to initiate leg movements.

c. The shuffling gait of PD causes the patient to be off balance and at risk for falling. Teaching the patient to use a wide stance with the feet apart, to lift the toes when walking, and to look ahead helps to promote a more balanced gait. Use of an elevated toilet seat and rocking from side to side will enable a patient to initiate movement. Canes and walkers are difficult for patients with PD to maneuver and may make the patient more prone to injury.

Which type of seizure occurs in children, is also known as a petit mal seizure, and consists of a staring spell that lasts for a few seconds? a. Atonic b. Simple focal c. Typical absence d. Atypical absence

c. The typical absence seizure is also known as petit mal and the child has staring spells that last for a few seconds. Atonic seizures occur when the patient falls from loss of muscle tone accompanied by brief unconsciousness. Simple focal seizures have focal motor, sensory, or autonomic symptoms related to the area of the brain involved without loss of consciousness. Staring spells in atypical absence seizures last longer than those in typical absence seizures and are accompanied by peculiar behavior during the seizure or confusion after the seizure.

The nurse is reinforcing teaching with a newly diagnosed patient with amyotrophic lateral sclerosis. Which statement would be appropriate to include in the teaching? a. "ALS results from an excess chemical in the brain, and the symptoms can be controlled with medication." b. "Even though the symptoms you are experiencing are severe, most people recover with treatment." c. "You need to consider advance directives now, since you will lose cognitive function as the disease progresses." d. "This is a progressing disease that eventually results in permanent paralysis, though you will not lose any cognitive function."

d Rationale: The disease results in destruction of the motor neurons in the brainstem and spinal cord, causing gradual paralysis. Cognitive function is maintained. Because there is no cure for amyotrophic lateral sclerosis (ALS), collaborative care is palliative and based on symptom relief. Death usually occurs within 3-6 years after diagnosis.

The nurse finds that an 87-year-old woman with Alzheimer's disease is continually rubbing, flexing, and kicking out her legs throughout the day. The night shift reports that this same behavior escalates at night, preventing her from obtaining her required sleep. The next step the nurse should take is to a. ask the physician for a daytime sedative for the patient. b. request soft restraints to prevent her from falling out of her bed. c. ask the physician for a nighttime sleep medication for the patient. d. assess the patient more closely, suspecting a disorder such as restless legs syndrome.

d Rationale: The severity of sensory symptoms of restless legs syndrome (RLS) ranges from infrequent, minor discomfort (paresthesias, including numbness, tingling, and "pins and needles" sensation) to severe pain. The discomfort occurs when the patient is sedentary and is most common in the evening or at night. The pain at night can disrupt sleep and is often relieved by physical activity, such as walking, stretching, rocking, or kicking. In the most severe cases, patients sleep only a few hours at night, which results in daytime fatigue and disruption of the daily routine. The motor abnormalities associated with RLS consist of voluntary restlessness and stereotyped, periodic, involuntary movements. The involuntary movements usually occur during sleep. Symptoms are aggravated by fatigue.

3. What is the most important method of diagnosing functional headaches? a. CT scan b. Electromyography (EMG) c. Cerebral blood flow studies d. Thorough history of the headache

d. The primary way to diagnose and differentiate between headaches is with a careful history of the headaches, requiring assessment of specific details related to the headache. Electromyelography (EMG) may reveal contraction of the neck, scalp, or facial muscles in tensiontype headaches but this is not seen in all patients. CT scans and cerebral angiography are used to rule out organic causes of the headaches.

22. The nurse explains to a patient newly diagnosed with MS that the diagnosis is made primarily by a. spinal x-ray findings. b. T-cell analysis of the blood. c. analysis of cerebrospinal fluid. d. history and clinical manifestations.

d. There is no specific diagnostic test for MS. A diagnosis is made primarily by history and clinical manifestations. Certain diagnostic tests may be used to help establish a diagnosis of MS. Positive findings on MRI include evidence of at least two inflammatory demyelinating lesions in at least two different locations within the central nervous system (CNS). Cerebrospinal fluid (CSF) may have increased immunoglobulin G and the presence of oligoclonal banding. Evoked potential responses are often delayed in persons with MS.


Ensembles d'études connexes

DECA Communication Skills Performance Indicators

View Set

Adult Health Exam 3 LaCharity delegation Practice questions

View Set

Lakota Unit 48-7 (Partitives: haŋké, húŋȟ)

View Set

F4 Manon des Sources-Vocabulaire (100% français)

View Set